Содержание

Тест по физике «Введение», 7 класс

Тест по физике №1

Тест по главе «Введение» и параграфам 1-6 учебника Физика.7 кл.: учеб. для общеобразоват. учреждений/ А.В. Перышкин. – 11-е изд., стререотип. – М.:Дрофа, 2007. – 192 с.: ил..

Диагностическая работа позволяет оценить достижение наиболее важных планируемых результатов в соответствии с содержанием курса физики 7 класса по главе «Введение».

Инструкция к тесту

Время прохождения теста — 15 минут. Вариант диагностической работы состоит из 11 заданий с выбором ответа. При прохождении теста необходимо выбрать один правильный вариант ответа в каждом вопросе. Задания 1 – 6, 11 оцениваются в 1 балл, 7 – 10 в 2 балла, и в 0 баллов, если нет ни одного элемента правильного ответа. Максимальный балл за всю работу составляет 15 баллов. Шкала перевода баллов в оценку: 13 – 15 баллов (5), 10 – 12 баллов (4), 7 – 9 баллов (3), 6 и менее баллов (2).

Демонстрационный вариант

1. Физическими явлениями называют?

а) любые превращения вещества,

б) любые проявления свойств вещества,

в) любые превращения вещества или проявления его свойств без изменения состава вещества,

г) любые превращения вещества или проявления его свойств, когда состав вещества изменяется.

2. Укажите физическое тело.

а) стекло, б) алюминий, в) дерево, г) окно.

3. Какое из представленных выражений обозначает физическую величину:

а) глубина реки, б) вода в реке, в) прозрачная вода, г) холодная вода.

4. В задании перечислены физические величины и единицы их измерения. Выберите верное соответствие.

а) время, метр (м), б) длина, секунда (с),

в) температура, миллиметр (мм), г) объем, кубический метр (м3).

5. В каком представленном ниже случае человек проводит опыт?

а) когда он сидит на берегу озера и глядит, как удаляется от него лодка,

б) когда он стоит на берегу и достает из воды упавшую в нее монету,

в) когда он бросает в реку гальку, щепки, кусочки бумаги и смотрит, какие из этих предметов утонут,

г) когда он наблюдает за облаками.

6. Ниже записаны численные обозначения приставок. В каком ответе допущена ошибка?

а) санти (с) – 0,01, б) милли (м) – 0,001,

в) микро (мк) – 10000, г) кило (к) – 1000.

7. Выразите 10 мл в см3, дм3, м3.

а) 100 см3; 0,01 дм3; 0,1 м3, б) 100 см3; 1000 дм3; 10000 м3,

в) 10 см3; 0,01 дм3; 0,00001 м3, г) 10 см3; 1 дм3; 0,01 м3.

8. Каковы будут значения длин, равных 4 м и 100 м, если их выразить в см.

а) 40 см и 10 см, б) 400 см и 1 см,

в) 400 см и 10 см, г) 40 см и 1 см.

9. Определите показания секундомера.

а) 42 мин 7 с, б) 49 с, в) 7 ч 42 мин, г) 7 мин 42 с.

10. На рисунке изображен карандаш, плотно обвитый тонкой проволокой, и линейка. Определите цену деления линейки и диаметр проволоки.

а) 1 мм; 2 мм. б) 1 см; 2 мм, в) 1,2 см; 3,6 см, г) 1 см; 2,4 см.

11. Почему каждому нужно знать физику?

а) потому что физика объясняет причины разных явлений природы,

б) эта наука позволяет создавать новую, все более совершенную технику,

в) потому что физика дает знания о самых общих законах природы, играющих большую роль в жизни каждого человека,

г) все ответы верны.

Правильные ответы

Номер вопроса

Ответ

Балл

1

в

1

2

г

1

3

а

1

4

г

1

5

в

1

6

в

1

7

г

2

8

в

2

9

а

2

10

а

2

11

г

1


 

Тест 1 в два варианта Введение 7 класс

1. Что изучает физика?

1) Явления, происходящие в неживой природе
2) Световые, тепловые, механические, звуковые, электрические и магнитные явления
3) Разные изменения в окружающем мире

3 Что из названного относится к веществам?

1) Вода 2) Самолет
3) Луна 4) Цветок

4. Когда ученику был задан вопрос: «Какие он знает вещества?», тот среди других назвал следующие: вода, железо, море, бумага. В каком случае ученик допустил ошибку?

1) вода 2) железо
3) море 4) бумага

5.  Гипотеза — это

1) обнаружение при наблюдении явления какой-либо закономерности
2) вывод из результатов опыта
3) предположение о том, как будет происходить то или иное явление

6. Что значит измерить физическую величину?

1) Сравнить её с другой величиной
2) Сравнить её с однородной величиной, принятой за единицу
3) Узнать, во сколько раз она меньше или больше величины, принятой за единицу

8. Какова цена деления мензурки, изображённой на рисунке?

1) 10 мл 3) 25мл

2) 12,5 мл 4) 50мл

Тест по физике Введение 7 класс

2 вариант

1. Физическое тело — это

1) любое твёрдое тело
2) предмет, который мы видим
3) тело, свойства которого изучаются в физике
4) любое тело в окружающем мире

2. Что из перечисленного относится к физическим телам?

1) Звук 2) Тепловоз
3) Пламя 4) Кислород

3. В каких случаях вещество, из которого может быть изготовлено тело, указано неправильно?

1) Лодка — пластмасса 2) Крыша — металл
3) Гвоздь — пластилин 4) Сумка — ткань

4. В каком из названных здесь случаев человек проводит опыт?

1) Когда он сидит на берегу озера и глядит, как удаляется от него лодка
2) Когда он бросает в реку гальку, щепки, кусочки бумаги и смотрит, какие из этих предметов утонут
3) Когда он стоит на берегу и достаёт из воды упавшую в неё монету

5. Как создаётся физическая теория?

1) Проводят наблюдения и опыты, устанавливают закономерности протекания физических явлений, выдвигают гипотезу об их причине
2) Ставят опыты, которые могут подтвердить или опровергнуть гипотезу
3) Подтвержденная гипотеза служит основанием для построения физической теории
4) Выполняются все пункты (1, 2, 3)

6. Какая единица длины (расстояния) принята как основная в международной системе единиц (СИ)?

1) Сантиметр 2) Метр
3) Километр 4) Миллиметр

7. Каковы будут значения длин, равных 4 м и 100 мм, если их выразить в сантиметрах?

1) 40 см и 10 см 2) 400 см и 10 см
3) 400 см и 1 см 4) 40 см и 1 см

8. Какова погрешность измерения секундомером, показанным на рисунке?

1) 10 с
2) 2 с
3) 1 с
4) 0,5 с

Ответы на тест по физике Введение
1-2
2-4
3-1
4-2
5-1
6-3
7-3
8-2
9-3
10-4
11-2
12-2
13-1
14-2
15-3
16-1
17-4
18-4

Тесты по физике. 7 класс. Тема: Введение. | Тест по физике (7 класс) по теме:

ВАРИАНТ 1.   Введение.

1. Какое из перечисленных физических явлений не является физическим?

а) гниение соломы,   б) нагревание воды,  

в) движение автомобиля,   г) электрический ток.

2. Укажите физическое тело.

а) дерево,   б) окно,   в) стекло,   г) алюминий.

3. Найдите ошибочное утверждение. Опыты проводят…

а) с определенной целью,   б) по обдуманному плану,  

в) не пользуясь приборами,   г) выполняя специальные измерения.

4. Какое из выражений обозначает физическую величину:

а) вода в реке,   б) глубина реки,   в) холодная вода,   г) прозрачная вода.

5. Ниже перечислены физические величины и единицы их измерения. Выберите верное соответствие.                                                                                                                              

а) длина, секунда (с),   б) время, метр (м),  

в) объем, кубический метр (м ),   г) температура, миллиметр (мм).

6. Записаны численные обозначения приставок. В каком примере допущена ошибка?

а) кило (к) – 1000,   б) микро (мк) – 10000,  

в) милли (м) – 0,001,   г) санти (с) – 0,01.

7. Выразите 10 мл в см  , дм  , м  .

а) 10 см  ; 1 дм  ; 0,01 м  ,   б) 10 см  ; 0,01 дм  ; 0,00001 м  ,  

в) 100 см  ; 1000 дм  ; 10000 м  ,   г) 100 см  ; 0,01 дм  ; 0,1 м  .

8. Укажите верхний предел измерения,

цену деления, объем жидкости в мензурке.

а) 150 см  ; 10 см  ; 80 см  ,

б) 200 см  ; 5 см  ; 90 см  ,

в) 200 см  ; 10 см  ; 80 см  ,

г) 200 см  ; 10 см  ; 90 см  .

9. Определите показания секундомера.

а) 7 ч 42 мин,   б) 7 мин 42 с,   в) 42 мин 7 с,   г) 49 с.

10. На рисунке в одном и том же объеме             а) 1 см; 2, 4 см,

изображены карандаш, плотно обвитый              б) 1,2 см; 3, 6 см,

тонкой проволокой, и линейка. Определите        в) 1 см; 2 мм,

цену деления линейки и диаметр проволоки.      г) 1 мм; 2 мм.

ВАРИАНТ 2.   Введение.

Какое из перечисленных физических явлений не относится к тепловым?

а) кипение воды,   б) таяние льда,  

в) горение свечи,   г) ледоход.

2. Укажите вещество.

а) чашка,   б) ваза,   в) фарфор,   г) стакан.

3. Ниже перечислены физические величины и приборы для их измерения. Выберите неверное соответствие.

а) длина, рулетка,   б) время, линейка,  

в) объем, мензурка,   г) температура, термометр.

4. Записаны численные обозначения приставок. В каком примере допущена ошибка?

а) милли (м) – 0,001,  б)   санти (с) – 0,01,

в) микро (мк) – 0,000001,  г) мега (М) – 0,00001.

5. Какое из приведенных значений может выражать объем жидкости?                                                                                                                              

а) 300 мл,   б) 100 дм,   в) 60 с,   г) 3 км.

6. Выразите 1,2 м в мм, см, дм.

а) 1200 мм; 12 см; 1,2 дм,   б) 120 мм; 1200 см; 120 дм,  

в) 120 мм; 120 см; 12 дм,   г) 1200 мм; 120 см; 12 дм.

7. Вычислите объем тела, выразите его в м  .          

а) 10 см  ; 0,1 м  ,   б) 36 см  ; 0,36 м  ,  

в) 36 см  ; 0,000036 м  ,   г) 48 см  ; 0,048 м  .

8. Определите верхний предел измерения,

цену деления мензурки, вместимость сосуда,

из которого вылили воду.

а) 50 см  ; 1 см  ; 23,5 см  ,

б) 50 см  ; 2 см  ; 20 см  ,

в) 50 мл  ; 2 мл  ; 20 мл  ,

г) 50 см  ; 1 см  ; 33,5 см  .

9. На рисунке изображен медицинский термометр. Определите нижний и верхний пределы измерения, цену деления, показания термометра.

а) 0-42 С; 0,1 С; 36,6 С,   б) 35-42 С; 0,1 С; 36,6 С,    

в) 0-35 С; 1 С; 36,6 С,   г) 35-42 С; 2 С; 38 С.    

10. На рисунке в одном и том же объеме             а) 1 см; 3 см,

изображены катушка с нитками                            б) 0,1 см; 1,5 мм,

и линейка. Определите  цену деления                  в) 0,1 мм; 15 мм,

линейки, толщину нитки.                                       г) 0,1 см; 0,15 см.

   Рис. 1

к учебнику А. В. Перышкина ОНЛАЙН


Чеботарева А. В. Тесты по физике. 7 класс: к учебнику А. В. Перышкина «Физика. 7 класс». ФГОС (к новому учебнику) / А. В. Чеботарева. — 12-е изд., перераб. и доп. — М., 2017. — 176 с. (Серия «Учебно-методический комплект»)
Данное пособие полностью соответствует федеральному государственному образовательному стандарту (второго поколения).
Пособие содержит тематические тестовые задания по физике для 7 класса, составленные к каждому параграфу учебника А. В. Перышкина «Физика. 7 класс». В издание также включены итоговые тематические работы: 6 контрольных тестов, каждый из которых представлен в четырех вариантах. Ко всем тестам даются ответы.
Пособие помогает осуществлять систематическую текущую проверку усвоения материала семиклассниками, своевременно выявлять пробелы в знаниях.
Издание адресовано как учителям физики, так и учащимся для самоконтроля.
СОДЕРЖАНИЕ
Предисловие …5
ВВЕДЕНИЕ …7
I. ПЕРВОНАЧАЛЬНЫЕ СВЕДЕНИЯ О СТРОЕНИИ ВЕЩЕСТВА …11
Молекулы. Движение молекул …11
Взаимодействие молекул. Агрегатные состояния вещества …13
1. Итоговый тест (темы «Введение» и «Первоначальные сведения о строении вещества») …15
Вариант I …15
Вариант II …19
Вариант III …22
Вариант IV …25
II. ВЗАИМОДЕЙСТВИЕ ТЕЛ (ЧАСТЬ 1) …29
Механическое движение …29
Скорость. Единицы скорости …30
Инерция. Взаимодействие тел …33
Масса тела …35
Плотность вещества …37
Расчет массы и объема тела по его плотности …39
2. Итоговый тест (темы «Механическое движение», «Масса тела», «Плотность вещества») …40
Вариант 1 …40
Вариант II …44
Вариант III… 47
Вариант IV …50
II. ВЗАИМОДЕЙСТВИЕ ТЕЛ (ЧАСТЬ 2)… 54
Сила. Явление тяготения. Сила тяжести …54
Сила упругости. Закон Гука …56
Вес тела …57
Единицы силы. Связь между силой тяжести и массой тела… 58
Сила тяжести на других планетах …60
Динамометр. Сложение двух сил, направленных по одной прямой …62
Сила трения …65
3. Итоговый тест (тема «Силы») …67
Вариант I …67
Вариант II …71
Вариант III …74
Вариант IV …78
III. ДАВЛЕНИЕ ТВЕРДЫХ ТЕЛ, ЖИДКОСТЕЙ И ГАЗОВ (ЧАСТЬ 1) …82
Давление. Единицы давления …82
Давление газа …84
Передача давления жидкостями и газами. Закон Паскаля …86
Давление в жидкости и газе …87
Расчет давления жидкости …89
Сообщающиеся сосуды …91
4. Итоговый тест (темы «Давление»,
«Давление в жидкости и газе», «Сообщающиеся сосуды») …93
Вариант I …93
Вариант II …96
Вариант III …98
Вариант IV …101
III. ДАВЛЕНИЕ ТВЕРДЫХ ТЕЛ, ЖИДКОСТЕЙ И ГАЗОВ (ЧАСТЬ 2) …104
Атмосферное давление …104
Измерение атмосферного давления …106
Манометры. Поршневой жидкостный насос. Гидравлический пресс …108
Действие жидкости и газа на погруженное в них тело …111
Архимедова сила …112
Плавание тел …114
Плавание судов. Воздухоплавание …116
5. Итоговый тест (темы «Атмосферное давление», «Архимедова сила», «Плавание тел») …119
Вариант I …119
Вариант II …122
Вариант III …126
Вариант IV …129
IV. РАБОТА И МОЩНОСТЬ. ЭНЕРГИЯ …133
Механическая работа. Единицы работы …133
Мощность. Единицы мощности …135
Простые механизмы. Рычаг …137
Момент силы. Применение рычагов …140
Блоки …142
«Золотое правило» механики …144
Центр тяжести тела. Условия равновесия тел …146
Коэффициент полезного действия механизма …148
Энергия …149
6. Итоговый тест (темы «Работа», «Мощность», «Энергия») …152
Вариант I …152
Вариант II …156
Вариант III …159
Вариант IV …162
ОТВЕТЫ …166
Приложение …175

Страница не найдена

Новости

9 авг

В одну из школ Нижневартовска на электронную почту поступило сообщение о минировании, заявил замглавы города Николай Лукаш.

9 авг

Заместитель председателя комитета Госдумы по информационной политике, информационным технологиям и связи Андрей Свинцов прокомментировал в беседе с RT сообщения о планах перевести госслужащих и сотрудников бюджетных организаций на российские мессенджеры и другие IT-сервисы.

9 авг

В России планируют перевести госслужащих и сотрудников бюджетных организаций на российские мессенджеры и другие IT-сервисы, пишет «Коммерсантъ» со ссылкой на план мероприятий госпрограммы «Создание дополнительных условий для развития отрасли информационных технологий». Документ Минцифры направлен в правительство.

6 авг

Выплаты на школьников в Московской области можно получить по одному заявлению в режиме онлайн на региональном сайте госуслуг.

6 авг

Власти Москвы планируют благоустроить территории 375 образовательных учреждений.

6 авг

Выпускница из Ростова-на-Дону Алика Осадчая, которая сдала единый государственный экзамен (ЕГЭ) на 400 баллов, поступила на юридический факультет МГИМО.

5 авг

Депутаты Госдумы, члены комитета по образованию и науке Михаил Берулава и Гаджимет Сафаралиев предложили сохранить практику онлайн-обучения после пандемии.

Тест по Физике «Введение. Строение вещества» 7 класс

«Введение. Строение вещества.»

І вариант.

  1. Выбрать правильный ответ

1. Длина, площадь, объем — это

а) …качества тела.

б) …его физические свойства.

в) …физические величины, характеризующие размеры тел.

г) Среди ответов нет верного.

2. К физическим телам относятся

а) …парта. в) …бензин.

б) …сахар. г) …комар.

3.Определите цену деления изображенной здесь шкалы прибора.

а) 30 ед.

б) 3 ед.

в) 6 ед.

г) 5 ед.

4.Мельчайшие частицы, из которых состоят различные вещества, называются…

а). атомами.

б). молекулами.

5. Все молекулы одного и того же вещества…

а). не отличаются друг от друга.

б). отличаются друг от друга.

6. При охлаждении объём тела…

а). уменьшается.

б). увеличивается.

7. Как зависит процесс диффузии от температуры?

а). Процесс диффузии замедляется с ростом температуры.

б). Процесс диффузии ускоряется с ростом температуры.

в). Процесс диффузии не зависит от изменения температуры

8. На расстояниях, сравнимых с размерами самих молекул (атомов)…

а). заметнее проявляются силы притяжения между молекулами, а при

дальнейшем сближении – силы отталкивания.

б). заметнее проявляются силы отталкивания между молекулами, а при

дальнейшем сближении — силы притяжения.

9. В каком состоянии может находиться ртуть?

а). Только в жидком.

б). В жидком, твердом и газообразном.

в). только в жидком.

10. Можно ли открытый сосуд заполнить газом на 40% его вместимости?

а). Да, можно.

б). Нет, нельзя.

в). Определенного ответа дать нельзя.

11. Вода замерзла и превратилась в лёд. Изменились ли при этом сами

молекулы воды?

а). Нет, не изменились.

б). Да, изменились.

в). Определённого ответа дать нельзя.

  1. Ответьте письменно.

  1. Опишите опыт, который подтверждает гипотезу о том, что вещества состоят из мельчайших частиц.

  2. Какова роль диффузии в природе?

ІІ вариант.

1.Выбрать правильный ответ

1.Физическое тело

а) медь б) стул в) движение г) снегопад

2. Физическое явление

а) свинец б) автомобиль в) кипение г) Луна

3. Цена деления линейки

а) 5см б) 60см в) 2,5см г) 10см

4. При нагревании объём тела…

а). увеличивается б). уменьшаются

5. Молекулы различных веществ…

а). не отличаются друг от друга б). отличаются друг от друга

6. Процесс диффузии присходит…

а). только в жидкостях и газах

б). только в жидкостях и твёрдых телах

в). в газах, жидкостях и твёрдых телах

7. Молекулы (атомы) притягиваются друг к другу. При этом они должны как бы слипнуться. Этого не происходит, потому что…

а). молекулы (атомы) непрерывно движутся

б). между молекулами (атомами) существуют силы отталкивания

8. Какие из указанных свойств принадлежат жидкостям?

а). легко изменяют форму, но сохраняют объём

б). не имеют собственной формы и постоянного объёма

в). Имеют собственную форму и объём

9. В каком состоянии может находиться чугун?

а). только в жидком

б). только в твёрдом

в). в твёрдом, жидком и газообразном

10. В бутылке находится вода объёмом 0,2 л. Её переливают в колбу вместимостью 0,5 л. Изменится ли объём воды?

а). уменьшится

б). не изменится

в). увеличится

11. В помещениях, где пользуются медицинским эфиром, обычно сильно пахнет. В каком состоянии находится эфир в этом помещении?

а). в твёрдом

б). в жидком

в). в газообразном

2.Ответьте письменно.

1

. Опишите опыт, который показывает изменение объема при нагревании жидкости

2.Почему плотность льда и воды различна?

Физика: уроки, тесты, задания.

Физика: уроки, тесты, задания.
    1. Введение. Макро- и микромир. Числа со степенью 10
    2. Наблюдения, опыты, измерения, гипотеза, эксперимент
    3. Физические величины. Международная система единиц
    1. Механическое движение. Траектория и путь
    2. Скорость. Неравномерное движение. Средняя скорость
    3. Что такое инерция
    4. Взаимодействие тел. Масса тела. Измерение массы тела на весах
    5. Плотность вещества. Связь массы, объёма тела с его плотностью
    6. Что такое сила. Сила гравитации. Сила тяжести
    7. Что такое вес тела. Свободное падение
    8. Измерение силы с помощью динамометра
    9. Деформации тел. Сила упругости. Закон Гука
    10. Взаимодействие тел. Сила трения
    1. Работа как физическая величина
    2. Мощность как характеристика работы
    3. Простые механизмы. Рычаг. Наклонная плоскость
    4. Подвижные и неподвижные блоки
    5. Полезная работа. Коэффициент полезного действия
    6. Энергия как физическая величина. Виды энергии
    1. Строение вещества. Молекулы и атомы
    2. Броуновское движение. Диффузия
    3. Притяжение и отталкивание молекул. Смачивание и капиллярность
    4. Изменение свойств веществ. Агрегатные состояния вещества
    1. Что такое давление и сила давления
    2. Давление твёрдых тел
    3. Давление газа. Применение сжатого воздуха
    4. Атмосферное давление и его измерение. Опыт Торричелли
    5. Давление в жидкости. Закон Паскаля
    6. Гидростатическое давление. Давление на дне морей и океанов
    7. Сообщающиеся сосуды. Водопровод. Шлюзы
    8. Гидравлический пресс. Насосы
    9. Закон Архимеда. Вес тела в жидкости
    10. Действие жидкости и газа на погружённое в них тело. Плавание тел
    11. Выталкивающая сила в газах. Воздухоплавание
  1. Класс заполнен на 100 %

    1. Тепловое движение. Связь температуры тела со скоростью движения молекул
    2. Внутренняя энергия. Два способа изменения внутренней энергии
    3. Виды теплопередачи
    4. Количество теплоты как физическая величина
    5. Что такое удельная теплоёмкость вещества
    6. Что такое удельная теплота сгорания топлива
    7. Закон сохранения энергии в механических и тепловых процессах
    1. Плавление и отвердевание тел. Температура плавления
    2. Что такое удельная теплота плавления
    3. Парообразование и конденсация
    4. Относительная влажность воздуха и её измерение. Психрометр
    5. Кипение. Температура кипения. Удельная теплота парообразования
    6. Объяснение изменений агрегатных состояний вещества
    7. Преобразования энергии в тепловых машинах
    8. Экологические проблемы использования тепловых машин
    1. Проводники, диэлектрики и полупроводники
    2. Взаимодействие заряженных тел. Электрическое поле
    3. Закон сохранения электрического заряда
    4. Дискретность электрического заряда. Электрон. Строение атомов
    5. Электрический ток. Электрическая цепь. Гальванические элементы
    6. Электрический ток в металлах. Полупроводниковые приборы
    7. Сила тока как физическая величина. Амперметр
    8. Электрическое напряжение как физическая величина. Вольтметр
    9. Электрическое сопротивление как физическая величина. Закон Ома
    10. Удельное сопротивление. Реостаты. Резисторы
    11. Последовательное и параллельное соединения проводников. Правила
    12. Понятия работы и мощности электрического тока
    13. Количество теплоты, выделяемое проводником с током
    14. Счётчик электрической энергии
    15. Виды ламп накаливания
    16. Расчёт электроэнергии, потребляемой бытовыми электроприборами
    17. Короткое замыкание. Электробезопасность. Плавкие предохранители
    1. Магнитное поле. Направление магнитных линий
    2. Свойства электромагнитов
    3. Постоянные магниты. Магнитное поле Земли
    4. Движение проводника в магнитном поле. Электродвигатель. Динамик и микрофон
    1. Источники света. Прямолинейность распространения света
    2. Понятие отражения света. Закон отражения. Плоское зеркало
    3. Понятие преломления света. Закон преломления
    4. Линза. Фокусное расстояние линзы. Построение изображений
    5. Оптическая сила линзы. Глаз как оптическая система. Оптические приборы
  1. Класс заполнен на 100 %

    1. Понятие материальной точки. Системы отсчёта
    2. Перемещение. Скорость прямолинейного равномерного движения
    3. Прямолинейное равноускоренное движение: мгновенная скорость, ускорение
    4. Графики зависимости величин от времени при равномерном движении
    5. Графики зависимости величин от времени при равноускоренном движении
    6. Графики зависимости кинематических величин от времени при равномерном и равноускоренном движении
    1. Относительность механического движения
    2. Первый закон Ньютона. Инерция. Инерциальные системы отсчёта
    3. Второй закон Ньютона. Сила трения скольжения
    4. Взаимодействие тел. Третий закон Ньютона
    5. Ускорение свободного падения. Изменение веса при движении
    6. Движение тела, брошенного вертикально вверх. Невесомость
    7. Закон всемирного тяготения. Гравитационная постоянная
    1. Понятие импульса тела
    2. Закон сохранения импульса. Виды взаимодействий
    3. Что такое реактивное движение
    1. Колебательное движение. Амплитуда, частота, период колебаний
    2. Колебательная система. Колебания груза на пружине. Математический маятник
    3. Превращение энергии при колебательном движении
    4. Вынужденные колебания. Резонанс
    5. Поперечные и продольные волны. Длина волны
    6. Звуковые волны. Скорость звука
    7. От чего зависят высота, тембр, громкость и резонанс звука
    1. Однородное и неоднородное магнитное поле
    2. Направление магнитных линий прямого проводника с током
    3. Как обнаружить магнитное поле. Правило левой руки
    4. Что такое индукция магнитного поля и магнитный поток
    5. Что такое электромагнитная индукция
    6. Направление индукционного тока. Правило Ленца. Явление самоиндукции
    7. Переменный ток. Генератор переменного тока
    8. Трансформатор. Передача электрической энергии на расстояние
    9. Электромагнитное поле. Скорость распространения электромагнитных волн
    10. Конденсатор. Колебательный контур. Принципы радиосвязи и телевидения
    11. Электромагнитная теория света
    12. Закон преломления света. Показатель преломления
    13. Дисперсия. Спектр. Типы оптических спектров
    14. Постулаты Бора. Поглощение и испускание света атомами. Линейчатые спектры
    1. Радиоактивность как свидетельство сложного строения атомов. Опыты Резерфорда
    2. Протонно-нейтронная модель ядра. Энергия связи частиц в ядре
    3. Альфа-, бета- и гамма-излучения. Радиоактивные превращения атомных ядер
    4. Методы наблюдения и регистрации частиц в ядерной физике
    5. Механизм деления ядер урана. Протекание цепной реакции
    1. Описание механического движения

По физике, 7 9 перышкин. Актуальный сборник задач по физике для семиклассников

М .: 2017 — 2 72с. М .: 2013 — 2 72с. М .: 2010 — 1 92с.

Учебное пособие полностью соответствует ФГОС (второе поколение). Сборник задач по физике А.В. Перышкина — необходимая составляющая учебного пособия по физике для 7–9 классов. Пособие ориентировано на учебники А.В. Перышкин «Физика. 7 класс», «Физика.8 класс »и учебник А.В. Перышкина и Е.М. Гутник« Физика. 9 класс ». Оно охватывает все разделы, изучаемые в 7–9 классах, поэтому окажет неоценимую помощь тем, кто изучает любые учебники физики из Федерального списка. Пособие основано на трудах А. В. Перышкина, изданных в разное время. , и представляет собой уникальный сборник из более чем 1800 задач по физике, который содержит задачи по каждому абзацу этих учебников А.В. Перышкина, а также ответы и справочный материал.Издание адресовано учителям физики, учащимся 7-9 классов, а также тем, кто готовится к Государственной итоговой аттестации по физике.

Формат: pdf ( 2017 г. , 19 изд., Перераб. И доп., 272с)

Размер: 5 Мб

Часы, скачать: drive.google ; Rghost

Формат: pdf ( 2013 г. , 9 изд., Перераб. И доп., 272с)

Размер: 3 Мб

Часы, скачать: проезд.Google ; Rghost

Формат: pdf ( 2010 г. , 5 изд., Стер., 192с)

Размер: 4.6 Мб

Часы, скачать: drive.google ; Rghost

СОДЕРЖАНИЕ
7 КЛАСС
ВВЕДЕНИЕ
1. Некоторые физические термины. Наблюдения и эксперименты 8
2. Физические величины. Измерение физических величин. 10
3. Точность и погрешность измерения 13
ИСХОДНАЯ ИНФОРМАЦИЯ О СТРУКТУРЕ ВЕЩЕСТВА
4.Строение вещества. Молекулы. Распространение в газах, жидкостях и твердых телах. Взаимное притяжение и отталкивание молекул 14
5. Агрегатное состояние вещества. Различие в молекулярной структуре твердых тел, жидкостей и газов 17
ВЗАИМОДЕЙСТВИЕ ТЕЛА
6. Механическое движение. Равномерное и неравномерное движение 18
7. Скорость. Единицы скорости. Расчет пути и времени движения 19
8. Инерция 26
9. Взаимодействие тел. Масса тела.Единицы массы. Измерение массы тела 27
10. Плотность вещества. Расчет массы и объема тела по его плотности 29
11. Прочность. Явление гравитации. Гравитация 34
12. Прочность эластичности. Закон Гука. Масса тела. Единицы силы 35
13. Связь между гравитацией и массой тела 38
14. Динамометр. Сложение двух сил, направленных по одной прямой. Результирующая сила 39
15. Сила трения. Трение покоя.Трение в природе и технике 44
ДАВЛЕНИЕ ТВЕРДЫХ ТЕЛ, ЖИДКОСТЕЙ И ГАЗОВ
16. Давление. Единицы давления. Способы понижения и повышения давления 47
17. Давление газа. Передача давления жидкостями и газами. Закон Паскаля 48
18. Давление в жидкости и газе. Расчет давления жидкости на дно и стенки сосуда 49
19. Сообщающиеся сосуды 53
20. Вес воздуха. Атмосферное давление. Опыт Торричелли. Гидравлические механизмы 54
21.Воздействие жидкости и газа на погруженное в них тело. Архимедова сила. Плавательный тел. Полеты на воздушном шаре 58
РАБОТА И ВЛАСТЬ. ENERGY
22. Механические работы. Единицы работы 63
23. Электроэнергия. Блоки питания 64
24. Рычаг. Баланс сил на рычаге. Момент силы. Рычаги в технике, быте и природе 67
25. Применение закона баланса рычагов к блоку. Золотое правило механики 69
26. Эффективность механизма 71
27.Энергия. Потенциальная и кинетическая энергия 72
28. Преобразование одного вида механической энергии в другой 76


8 СТУПЕНЬ
ТЕПЛОВЫЕ ЯВЛЕНИЯ
29. Тепловое движение. Температура. Внутренняя энергия …. 78
30. Способы изменения внутренней энергии тела. Теплопроводность. Конвекция. Излучение 79
31. Количество тепла. Единицы тепла. Удельная теплоемкость. Расчет количества тепла, необходимого для нагрева тела или выделяемого им при охлаждении 82
32.Энергетическое топливо. Удельная теплотворная способность 86
33. Закон сохранения и преобразования энергии в механических и тепловых процессах 87
ИЗМЕНЕНИЕ АГРЕГАТНЫХ СОСТОЯНИЙ ВЕЩЕСТВА
34. Агрегатные состояния вещества. Плавление и упрочнение кристаллических тел. График плавления и затвердевания кристаллических тел. Удельная теплота плавления 90
35. Испарение. Поглощение энергии при испарении жидкости и ее выделение при конденсации пара.Кипячение. Удельная теплота испарения и конденсации 94
36. Влажность 97
37. Работа газа и пара при расширении. Двигатель внутреннего сгорания. Паровая турбина. КПД теплового двигателя 99
ЭЛЕКТРИЧЕСКИЕ ЯВЛЕНИЯ
38. Электрификация соприкасающихся тел. Взаимодействие заряженных тел. Два вида обвинений. Электроскоп. Проводники и непроводники электричества. Электрическое поле. Делимость электрического заряда. Электрон. Строение атомов.Объяснение электрических явлений 102
39. Электрический ток. Источники электрического тока. Электрическая схема и ее составные части. Электрический ток в металлах. Воздействие электрического тока. Направление электрического тока 106
40. Сила тока. Единицы тока. Амперметр. Измерение тока 108
41. Электрическое напряжение. Единицы напряжения. Вольтметр. Измерение напряжения. Зависимость тока от напряжения. Закон Ома по статье 109
42. Электрическое сопротивление проводников.Единицы сопротивления. Расчет сопротивления проводника. Удельное сопротивление 111
43. Последовательное соединение проводов 115
44. Параллельное соединение проводов 119
45. Работа и мощность электрического тока. Единицы электрического тока. Нагревательные проводники электрическим током. Закон Джоуля-Ленца 126
ЭЛЕКТРОМАГНИТНЫЕ ЯВЛЕНИЯ
46. Магнитное поле. Магнитные линии. Магнитное поле Земли. Электромагниты. Постоянные магниты. Действие магнитного поля на проводник с током 133
ЯВЛЕНИЯ СВЕТА
47.Источники света. Распространение света 139
48. Отражение света. Закон отражения света.
Плоское зеркало 140
49. Преломление света. Закон преломления света 143
50. Линзы. Оптическая сила линзы. Изображения, полученные объективом 147


9 КЛАСС
ЗАКОНЫ ВЗАИМОДЕЙСТВИЯ И ДВИЖЕНИЯ ТЕЛ
51. Материальная точка. Справочная система. Движущийся. Определение координат движущегося тела 154
52. Движение с прямолинейным равномерным движением 157
53.Прямолинейное равноускоренное движение. Ускорение, скорость, движение 161
54. Относительность движения. Инерциальные системы отсчета. Первый закон Ньютона 167
55. Второй закон Ньютона 172
56. Третий закон Ньютона 179
57. Свободное падение тел. Движение тела, брошенного вертикально вверх 184
58. Закон всемирного тяготения. Ускорение свободного падения на Земле и других небесных телах 186
59. Прямолинейное и криволинейное движение. Движение тела по кругу с постоянным модулем скорости.Искусственные спутники Земли 188
60. Импульсное тело. Закон сохранения количества движения 194
МЕХАНИЧЕСКИЕ КОЛЕБАНИЯ И ВОЛНЫ. ЗВУК
61. Колебания и волны 200
ЭЛЕКТРОМАГНИТНОЕ ПОЛЕ
62. Направление тока и направление линий его магнитного поля. Правило левой руки. Индукция магнитного поля 208
63. Явление электромагнитной индукции. Правило Ленца. Явление самоиндукции.Трансформатор 211
64. Электромагнитное поле. Электромагнитные волны. Конденсатор. Колебательный контур 216
СТРОЕНИЕ АТОМА И АТОМНОГО ЯДРА. ИСПОЛЬЗОВАНИЕ ЭНЕРГИИ АТОМНЫХ ЯДЕР
65. Радиоактивные превращения атомных ядер. Строение и состав атомного ядра. Энергия общения. Дефект массы 219
Ответы 223
Таблицы физических величин 237
Литература 269

Если говорить глобально, физику можно отнести к науке об окружающем мире, о природе.Она изучает как материальные, так и энергетические аспекты Вселенной, объясняет и логически обосновывает различные явления, происходящие в природе. Эта наука лежит в основе технического прогресса общества. А чтобы полно и качественно донести до школьников 7–9 классов основы физики, известные специалисты В.И. Лукашик, Иванова Е.В. создал книгу для учебника физики.

Основные функции ГДЗ:

1. формирование аналитических способностей ребенка путем постоянного самоанализа выполненного домашнего задания и анализа выявленных ошибок;

2.помощь ученикам в правильном решении экспериментальных задач и задач с различными неполными ответами, которые помогают ребенку творчески выразить себя;

3. Самостоятельный разбор семиклассниками задач на новую тему из абзаца;

4. Качественная подготовка студентов к предстоящей аудиторной работе и к любому контролю знаний;

5. закрепить изученную тему, решая повторные задания;

6. Отличный справочный материал для старшеклассников, готовящихся к экзаменам и итоговому тестированию;

7.дать родителям возможность проверить уровень подготовленности своего ребенка и помочь ему с домашним заданием;

8. Дополнительный материал для учителей, помогающий качественно и подробно разработать план будущего урока.

Сборник ГДЗ по физике для 7–9 классов Сборник заданий Лукашик предполагает изучение материала курса физики для 7, 8 и 9 классов. Поэтому первый раздел пособия именуется ИСХОДНАЯ ИНФОРМАЦИЯ О ФИЗИЧЕСКИХ ТЕЛАХ И ИХ СВОЙСТВА.В нем семиклассники поймут измерение физических величин и узнают о структуре материи. Ученики решают все задачи по законам движения молекул и температуры тела. Второй раздел посвящен движению и взаимодействию тел. Здесь студенты узнают о равномерном и неравномерном прямолинейном движении, знакомятся с понятием инерции тел. Будут решены проблемы определения плотности материи. Затронута тема явления гравитации и гравитации.Авторы восьмиклассников вводят законы Ньютона. Также студенты могут легко изобразить мощность графически. Третий раздел, озаглавленный «Давление твердых тел, газов и жидкостей», предоставляет восьмиклассникам формулы для решения задач, связанных с давлением. Этот раздел предполагает параграф о подвижности частиц жидкостей и газов. Все задачи из учебника по закону Паскаля будут решены. Не останутся без внимания и работы на сообщающихся сосудах. И Закон Архимеда перестанет быть препятствием для хорошей успеваемости учащихся.Также здесь будет рассказано о характеристиках манометров и насосов. Следующий раздел, посвященный работе и мощности, также включает изучение простых механизмов и энергии. Авторы дадут все формулы для нахождения работы и мощности, объяснят суть рычагов и блоков, помогут рассчитать КПД механизмов и энергии. Также сюда включен параграф о балансе тел.

В пятом разделе рассказывается о механической вибрации и волне. Шестое предполагает анализ тепловых явлений.В нем представлены все виды теплообмена, приведены формулы для измерения количества тепла. Знакомые девятиклассники с понятиями плавления и закалки. Также ученики решат проблему испарения и кипячения. В шестом разделе не будут оставлены без внимания тепловые машины и влажность воздуха. Седьмой раздел полностью посвящен электрическим явлениям. Ученикам необходимо запомнить множество различных формул, а именно формулу силы тока, напряжения и сопротивления. Студенты в лаборатории могут легко собрать электрическую схему и рассчитать задания по закону Ома.Они узнают все об электромагнитных явлениях и тепловом эффекте тока.

Следующий раздел посвящен световым явлениям, где подробно рассматриваются упражнения по распространению света, его отражению и преломлению. Девятиклассники узнают о плоском зеркале и знакомятся с разными линзами. А по окончании курса физики для девятого класса ученики должны будут пройти раздел, посвященный строению атома и атомного ядра. Он включает параграфы о радиоактивном распаде, ядерных реакциях и элементарных частицах.Такой материал полностью соответствует ФГОС и рекомендуется учащимся общеобразовательных школ при подготовке к тестам и олимпиадам.

Решатель физики для 7–9 классов Перышкин представляет собой сборник готовых домашних заданий, выполненных упражнений с пояснениями и комментариями. Он составлен на основе сборника задач, разработанного А. П. Перышкиным, и содержит правильно выполненные упражнения по всем вопросам школьного курса физики.

Решатель по физике 7-9 классы сборника задач Пёрышкин — практическая помощь в изучении предмета

Ключевым моментом при изучении физики является понимание студентом физических явлений и процессов на практике.В связи с этим особую актуальность приобретает специальная практическая задача при изучении предмета, которая содержит не только готовые ответы на более чем 1870 заданий и упражнений, но и подробные комментарии и инструкции по их выполнению.

Если ученик будет использовать ГДЗ по физике для 7-9 классов Перышкина в процессе подготовки домашнего задания, то он быстро научится понимать задания по предмету, что положительно повлияет на его текущую успеваемость и поможет успешно сдать экзамен. итоговая аттестация.Решатель позволяет:

  • понимать алгоритм решения проблем;
  • разбираться в специфике применения формул и законов;
  • развивать логическое и абстрактное мышление.

Самый удобный способ использовать решатель физики — через наш сайт. Его интерфейс разработан для максимального удобства пользователя и экономии времени:

  1. вы можете получить доступ к ресурсу со своего телефона, ноутбука или планшета;
  2. для того, чтобы найти нужный вам учебник, достаточно указать имя или ФИО автора в строке поиска;
  3. по номеру упражнения, ответ легко найти в таблице выше.

Стоит добавить, что база резольверов на сайте регулярно обновляется, что позволяет студентам представлять наиболее актуальные решения задач по физике. Для индивидуальных упражнений сразу представлено несколько решений из разных сборников Госдумы.

ГДЗ по физике 7-9 классы Порышкина — сборник задач 2013-2017 гг.

Задачи и упражнения по физике для 7–9 классов были объединены А. Перышкиным в единый сборник, выпущенный в 2013 году издательством «Экзамен».

Практическое пособие состоит из трех разделов — по классам школьной программы. В каждом из них задания разделены на главы — в соответствии с учебным планом:

  • класс 7 — строение веществ, взаимодействие тел, давление, работа, сила, энергия;
  • 8 класс — тепловые явления и агрегатное состояние веществ, электрические явления, электромагнитные явления, световые явления;
  • 9 класс — законы взаимодействия и движения тела, механические колебания и волны, звук, электромагнитное поле, строение атома, использование ядерной энергии.

Исследователь к сборнику задач по физике, подготовленному А.В. Перышкина, Предназначен для учащихся седьмых, восьмых и девятых классов общеобразовательных школ России. Пособие содержит множество примеров, в которых подробно объясняется, как решать различные типы задач по физике. Конечно, с таким учебником стало намного проще и быстрее, чем раньше, и результат не заставит себя ждать.

Методическая характеристика сборника

Приведенные выше понятия, формулы и описанные явления природы соответствуют перечню, приведенному в Федеральном государственном образовательном стандарте (Федеральном образовательном стандарте).Упражнения могут быть полезны учителям и наставникам при составлении программ авторских работ по курсу этого предмета. Готовые домашние задания онлайн развивают самостоятельность, мотивируют осознанный подход к обучению и помогают ученику быстро достичь необходимых целей:

  • подготовиться к самостоятельной и контрольной работе , а также пройти внешнее тестирование, чтобы узнать свой объективный прогресс.
  • для обучения по программе общеобразовательной школы, одновременно обучаясь на дому.
  • , чтобы повторить пройденный материал и лучше понять закономерности и исключения из практических правил, научиться независимо использовать фундаментальные законы природы для разработки собственных теорий.
  • в короткие сроки списать домашнее задание.
  • , чтобы сформировать культуру строгого физического и математического мышления.

В 7–9 классах сдается большой объем материала, который касается практически всех основных разделов физики. Это механика (включая статику, динамику и кинематику), молекулярная физика (включая идеи о тепловом движении атомов и молекул, а также свойства идеального газа), электростатика и электродинамика, основные оптические явления и идеи о магнитах. и магнетизм.По окончании курса студенты знакомятся с принципами ядерной физики.

Пособие А.В. Перышкин с правильными ответами написано с учетом всех учащихся общеобразовательных школ. Таким образом, его целевая аудитория — все дети соответствующего возраста.

ГДЗ физика 7 9 перышкин. Как работает пособие?

Учебное пособие полностью соответствует ФГОС (второе поколение).Сборник задач по физике А. В. Перышкина является необходимой составной частью учебного пособия по физике для 7–9 классов. Пособие ориентировано на учебники А.В. Перышкина «Физика. 7 класс »,« Физика. 8 класс »и учебник А. В. Перышкина и Е. М. Гутника« Физика. 9 класс. «Оно охватывает все разделы, изучаемые в 7–9 классах, поэтому окажет неоценимую помощь тем, кто изучает любые учебники физики из Федерального списка».
Пособие основано на трудах А.В. Перышкина, изданные в разное время и представляют собой уникальный сборник, включающий более 1800 задач по физике. Сборник содержит задания по каждому абзацу этих учебников А. В. Перышкина, а также ответы и справочный материал. Издание адресовано учителям физики, учащимся 7-9 классов, а также тем, кто готовится к основному государственному экзамену по физике. Приказом Минобрнауки России № 699 учебники издательства «Экзамен» допущены к использованию в общеобразовательных организациях.

Строение вещества. Молекулы. Броуновское движение. Распространение в газах, жидкостях и твердых телах. Взаимное притяжение и отталкивание молекул.
Камни не очень сжимаются, но металлы (даже очень плотные) можно сжать до 0,75 от первоначального объема с помощью мощного пресса. Почему возможно такое сильное сжатие?
Воду налили в стеклянную бутылку и поместили в морозильную камеру. Что будет с бутылкой и почему?
Меняется ли емкость кровеносных сосудов при изменении их температуры?
Отличаются ли молекулы воды в горячем чае от молекул воды в холодном лимонаде?
Стоит ли наливать полный чайник воды, если нужно в нем вскипятить воду?
Как называется физическое явление, благодаря которому можно мариновать овощи на зиму? Как соль переходит из воды в овощи при засолке?
Плотно закрученную крышку банки легче открутить, если она нагрета.Почему?
Если переместить надутый шар с тепла на холод, что произойдет с его объемом? Почему?
Горячие стеклянные стаканы не рекомендуется вставлять друг в друга. Почему?
Почему сложенные вместе стекла трудно разделить?
Надавите на две деревянные линейки с усилием. Легко ли их разделить? Объясните наблюдаемое явление.


Скачать бесплатно электронную книгу в удобном формате, смотреть и читать:
Скачать книгу Сборник задач по физике 7-9 классы, Перышкин А.В., 2017 — fileskachat.com, быстрая и бесплатная загрузка.

  • Сборник задач по физике, К учебникам А.В. Перышкина, 7-9 классы, А. Перышкин, 2013

Следующие учебники и книги:

  • Сборник контрольных работ по физике для заочников 10 класса, Коваленкова О.В., Лобач Д.И., Малашонок В.А., Развина Т.И., Ракина Н.Н., 2004 г.
  • Сборник контрольных работ по физике для заочников 9 класса, Коваленкова О.В., Лобач Д.И., Малашонок В.А., Развина Т.И., Ракина Н.Н., 2004

ГДЗ Физика 7-9 классы Сборник задач

Хотите познать азы такой сложной науки, как физика, без лишних мучений и долгих тусовок по учебникам? Не можете правильно решить задачу , а завтра брать домашнее задание? Устали регулярно получать двойки по нелюбимой теме? В любой из этих ситуаций я готов помочь облачить ГДЗ на 7, 8, 9 классы по набору заданий от А.В. Перышкина, , что наконец-то позволит получать удовольствие от учебы и значительно повысить успеваемость.

Как работает пособие?

Используйте компиляцию с правильными ответами может быть для двух основных целей:

  • если вам не нравится физика и вы решили связать свое будущее с гуманитарными науками, то вы можете просто списать готовые решения, составленные в соответствии со стандартами ФГОС, и получить отличные оценки как за домашнее задание, так и за для контроля.
  • , и если вы все еще любите физику и хотите улучшить свои знания, то эта книга поможет вам сделать это быстрее всего. Решебник — отличное пособие для самостоятельного изучения. Сначала прочтите тему в руководстве, затем выполните несколько задач и разберите их, используя сборник. Такой подход позволит разобраться даже с тем разделом, который в течение многих недель оставался непонятным.

Электронное руководство, автор которого А.В. Перышкин должен быть доступен каждому, кто является родителем ученика средней школы.Ведь книга оказывает огромную помощь не только детям, но и взрослым, которые отдают предпочтение контрольному домашнему заданию и помогают своему ребенку решать трудности. Теперь вам не нужно краснеть каждый раз, когда молодой ученик сталкивается с трудной задачей, и вы просто не помните школьную программу. В резольвере есть подробные ответы на все вопросы из соответствующего учебника, а это значит, что благодаря пояснениям вам будет очень легко вспомнить давно забытый материал.

Использовать GDZ предельно просто: достаточно сделать пару кликов на Интернет-страницах, потому что — это онлайн-доступ . Теперь учеба станет легким и увлекательным занятием абсолютно для каждого студента.

Каждый человек знаком с законами физики, потому что физические явления находятся рядом с нами повсюду и пренебрегать ими невозможно. Каждый человек измеряет температуру окружающей среды, включает электроприборы, пользуется Интернетом. Но во всех этих манипуляциях лежит знание такой науки, как физика.Ассистентом в освоении этого предмета для 7-9 классов был замечательный специалист своего дела А.В. Пёрышкин, подготовивший книгу для школьного учебника физики.

Автор не только полностью и легко раскрывает все упражнения из учебника, но и добавляет к готовым ответам необходимые комментарии, которые помогают лучше разобраться в непонятной теме из абзаца. Используя GDZ, ученик откроет для себя различные алгоритмы решения задач и узнает, как их использовать при выполнении домашних и классных заданий.Также семиклассники научатся правильно составлять свой раствор и смогут разобраться со всеми лабораторными работами.

Готовые ответы на сборник задач для 7–9 классов полностью соответствуют ФГОС и рекомендуются учащимся, готовящимся к тестам или олимпиадам. ГДЗ по физике включают подробный анализ решений более 1800 задач. Материал представлен в достаточно доступной форме, которая будет понятна студенту любого уровня подготовки. ГДЗ по физике для 7–9 классов Сборник задач Порышкин раскрывает суть школьных учебников Перышкина для 7, 8 и 9 классов.

Пособие разделено на части, соответствующие седьмому, восьмому и девятому годам обучения. Седьмой класс предполагает изучение таких тем, как строение материи; телесное взаимодействие; давление твердых тел, жидкостей и газов; работа, сила и энергия. Семиклассникам предстоит разобраться с основами механического движения, научиться рассчитывать путь и время движения.Они также научатся решать задачи на единицу массы и рассчитают плотность веществ. Студенты познакомятся с феноменом гравитации и гравитацией. Без труда они смогут решить все задачи по закону Гука. Семиклассники узнают о динамометре и смогут складывать две силы, направленные по одной прямой. А по окончании курса физики для седьмого класса ученики выучат параграф о силе трения.

Восьмой класс принесет знания по таким разделам, как: тепловые явления и возможные изменения агрегатного состояния вещества; электрические и электромагнитные явления; интересные световые явления.Восьмиклассники получат знания о тепловом движении, теплопроводности, конвекции и излучении. Ученики решают все задачи по закону сохранения и преобразования энергии в механических и тепловых процессах. Затем они познакомятся с проводниками и непроводниками электричества. Студенты поймут и запомнят формулы силы тока, напряжения и сопротивления, работы и мощности. В 8-м классе будет затронута тема магнитного поля Земли и обсужден вопрос об источнике света и законе преломления.

Девятый класс включает в школьную программу параграфы по темам: законы взаимодействия и движения тел; механические волны и колебания, а также звук; электромагнитные поля; строение атома и его ядра. Первоначально девятиклассники будут иметь дело с материальными пунктами и системой координат. Затем они будут решать задачи по темам ускорения, скорости и движения. Они разработают первый и второй законы Ньютона, займутся прямолинейным и криволинейным движением.Студенты изучат основы вибраций и волн. Они смогут научиться распознавать направление тока и направление линий его магнитного поля. А по окончании курса девятиклассники поймут, как использовать энергию атомных ядер. ГДЗ способствуют проявлению самостоятельности ребенка, также они развивают аналитические способности и логическое мышление. Пособие дает знания теорий и законов, учит правильной последовательности решения задач.Такой сборник поможет школьникам значительно повысить успеваемость и не боится контролировать знания в школе.

ГДЗ к учебнику по физике для 7-х классов Порышкин А.В. можно скачать.

Сборник заданий «Физика, 7, 8, 9 класс» (полезен практически каждому современному школьнику и значительно повысит уровень понимания и усвоения следующих разделов программы:

  • механика,
  • термическая теория
  • электростатика и электродинамика,
  • оптика и ее наиболее интересные явления,
  • избранных разделов магнетизма
  • начальное понимание ядерных процессов.

Характеристики преимущества

Готовые домашние задания онлайн позволяют заниматься как самостоятельно, так и под присмотром старших членов семьи. Благодаря этой функции подготовка ребенка к урокам стала намного проще. В выкройке много качественных иллюстраций, рисунков, а также информативных схем. Они объясняют сложные задачи и методы преобразования математических выражений для получения физически значимого результата.

Авторы приложили значительные усилия к тому, чтобы содержание книги полностью соответствовало федеральным государственным образовательным стандартам (ФГОС). Сборник с правильными ответами может успешно использоваться в произвольной общеобразовательной школе страны. Формирует у детей прочный фундамент знаний и умений, умение самостоятельно определять и устранять текущие проблемы в обучении. Материалы разделены по уровню сложности.

Акцент делается на кинематике и правильном решении задач на эту тему (движение двух тел, понятие ускорения, поступательное движение).После этого происходит броуновское движение, рассматриваются законы для идеального газа и на основе основных представлений выводятся некоторые практически важные и приближенные формулы для описания многоатомных систем. Ученики работают с понятиями давления и плотности, учатся различать агрегатные состояния вещества. Учебник помогает:

  • чувствую себя увереннее.
  • понимать принципы поиска ответов на вопросы о природе.
  • заинтересовался физикой на продвинутом уровне.

Онлайн-руководство A.V. Также Перышкин можно использовать при подготовке к ОГЭ.

М .: 2017 — 2 72с. М .: 2013 — 2 72с. М .: 2010 — 1 92с.

Учебное пособие полностью соответствует ФГОС (второе поколение). Сборник задач по физике А.В. Перышкина — необходимая составляющая учебного пособия по физике для 7-9 классов. Пособие ориентировано на учебники А.В. Перышкин «Физика. 7 класс», «Физика.8 класс »и учебник А.В. Перышкина и Е.М. Гутник« Физика. 9 класс ». Оно охватывает все разделы, изучаемые в 7–9 классах, поэтому окажет неоценимую помощь тем, кто изучает любые учебники физики из Федерального списка. Пособие основано на трудах А.В. Перышкина, изданных в разное время. , и представляет собой уникальный сборник из более чем 1800 задач по физике, который содержит задачи по каждому абзацу этих учебников А.В. Перышкина, а также ответы и справочный материал.Издание адресовано учителям физики, учащимся 7-9 классов, а также тем, кто готовится к Государственной итоговой аттестации по физике.

Формат: pdf ( 2017 г. , 19 изд., Перераб. И доп., 272с)

Размер: 5 Мб

Часы, скачать: drive.google ; Rghost

Формат: pdf ( 2013 г. , 9-е изд., Перераб. и доп., 272с)

Размер: 3 Мб

Часы, скачать: проезд.Google ; Rghost

Формат: pdf ( 2010 г. , 5 изд., Стер., 192с)

Размер: 4.6 Мб

Часы, скачать: drive.google ; Rghost

СОДЕРЖАНИЕ
7 КЛАСС
ВВЕДЕНИЕ
1. Некоторые физические термины. Наблюдения и эксперименты 8
2. Физические величины. Измерение физических величин. 10
3. Точность и погрешность измерения 13
ИСХОДНАЯ ИНФОРМАЦИЯ О СТРУКТУРЕ ВЕЩЕСТВА
4.Строение вещества. Молекулы. Распространение в газах, жидкостях и твердых телах. Взаимное притяжение и отталкивание молекул 14
5. Агрегатное состояние вещества. Различие в молекулярной структуре твердых тел, жидкостей и газов 17
ВЗАИМОДЕЙСТВИЕ ТЕЛА
6. Механическое движение. Равномерное и неравномерное движение 18
7. Скорость. Единицы скорости. Расчет пути и времени движения 19
8. Инерция 26
9. Взаимодействие тел. Масса тела. Единицы массы.Измерение массы тела по шкале 27
10. Плотность вещества. Расчет массы и объема тела по его плотности 29
11. Прочность. Явление гравитации. Гравитация 34
12. Прочность эластичности. Закон Гука. Масса тела. Единицы мощности 35
13. Связь между силой тяжести и массой тела 38
14. Динамометр. Сложение двух сил, направленных по одной прямой. Результирующая сила 39
15. Сила трения. Трение покоя. Трение в природе и технике 44
ДАВЛЕНИЕ ТВЕРДЫХ ТЕЛ, ЖИДКОСТЕЙ И ГАЗОВ
16.Давление. Единицы давления. Способы понижения и повышения давления 47
17. Давление газа. Передача давления жидкостями и газами. Закон Паскаля 48
18. Давление в жидкости и газе. Расчет давления жидкости на дно и стенки сосуда 49
19. Сообщающиеся сосуды 53
20. Вес воздуха. Атмосферное давление. Опыт Торричелли. Гидравлические механизмы 54
21. Воздействие жидкости и газа на погруженные в них тела. Архимедова сила. Плавательный тел. Полеты на воздушном шаре 58
РАБОТА И ВЛАСТЬ.ENERGY
22. Механические работы. Единицы работы 63
23. Электроэнергия. Блоки питания 64
24. Рычаг. Баланс сил на рычаге. Момент силы. Рычаги в технике, быте и природе 67
25. Применение закона баланса рычагов к блоку. Золотое правило механики 69
26. КПД механизма 71
27. Энергия. Потенциальная и кинетическая энергия 72
28. Преобразование одного вида механической энергии в другой 76


8 СТУПЕНЬ
ТЕПЛОВЫЕ ЯВЛЕНИЯ
29.Тепловое движение. Температура. Внутренняя энергия …. 78
30. Способы изменения внутренней энергии тела. Теплопроводность. Конвекция. Излучение 79
31. Количество тепла. Единицы тепла. Удельная теплоемкость. Расчет количества тепла, необходимого для нагрева тела или выделяемого им при охлаждении 82
32. Энергетическое топливо. Удельная теплотворная способность 86
33. Закон сохранения и преобразования энергии в механических и тепловых процессах 87
ИЗМЕНЕНИЕ СОСТОЯНИЯ ВЕЩЕСТВА
34.Агрегированные состояния вещества. Плавление и упрочнение кристаллических тел. График плавления и затвердевания кристаллических тел. Удельная теплота плавления 90
35. Испарение. Поглощение энергии при испарении жидкости и ее выделение при конденсации пара. Кипячение. Удельная теплота испарения и конденсации 94
36. Влажность 97
37. Работа газа и пара при расширении. Двигатель внутреннего сгорания. Паровая турбина. КПД теплового двигателя 99
ЭЛЕКТРИЧЕСКИЕ ЯВЛЕНИЯ
38.Электрификация контактирующих тел. Взаимодействие заряженных тел. Два вида обвинений. Электроскоп. Проводники и непроводники электричества. Электрическое поле. Делимость электрического заряда. Электрон. Строение атомов. Объяснение электрических явлений 102
39. Электрический ток. Источники электрического тока. Электрическая схема и ее составные части. Электрический ток в металлах. Воздействие электрического тока. Направление электрического тока 106
40. Сила тока. Единицы тока.Амперметр. Измерение тока 108
41. Электрическое напряжение. Единицы напряжения. Вольтметр. Измерение напряжения. Зависимость тока от напряжения. Закон Ома по статье 109
42. Электрическое сопротивление проводников. Единицы сопротивления. Расчет сопротивления проводника. Удельное сопротивление 111
43. Последовательное соединение проводов 115
44. Параллельное соединение проводов 119
45. Работа и мощность электрического тока. Единицы электрического тока. Нагревательные проводники электрическим током.Закон Джоуля-Ленца 126
ЭЛЕКТРОМАГНИТНЫЕ ЯВЛЕНИЯ
46. Магнитное поле. Магнитные линии. Магнитное поле Земли. Электромагниты. Постоянные магниты. Действие магнитного поля на проводник с током 133
ЯВЛЕНИЯ СВЕТА
47. Источники света. Распространение света 139
48. Отражение света. Закон отражения света.
Плоское зеркало 140
49. Преломление света. Закон преломления света 143
50.Линзы. Оптическая сила линзы. Изображения, полученные объективом 147


9 КЛАСС
ЗАКОНЫ ВЗАИМОДЕЙСТВИЯ И ДВИЖЕНИЯ ТЕЛ
51. Материальная точка. Справочная система. Движущийся. Определение координат движущегося тела 154
52. Движение с прямолинейным равномерным движением 157
53. Прямолинейное равноускоренное движение. Ускорение, скорость, движение 161
54. Относительность движения. Инерциальные системы отсчета. Первый закон Ньютона 167
55.Второй закон Ньютона 172
56. Третий закон Ньютона 179
57. Свободное падение тел. Движение тела, брошенного вертикально вверх 184
58. Закон всемирного тяготения. Ускорение свободного падения на Земле и других небесных телах 186
59. Прямолинейное и криволинейное движение. Движение тела по кругу с постоянным модулем скорости. Искусственные спутники Земли 188
60. Импульсное тело. Закон сохранения количества движения 194
МЕХАНИЧЕСКИЕ КОЛЕБАНИЯ И ВОЛНЫ.ЗВУК
61. Колебания и волны 200
ЭЛЕКТРОМАГНИТНОЕ ПОЛЕ
62. Направление тока и направление линий его магнитного поля. Правило левой руки. Индукция магнитного поля 208
63. Явление электромагнитной индукции. Правило Ленца. Явление самоиндукции. Трансформатор 211
64. Электромагнитное поле. Электромагнитные волны. Конденсатор. Колебательный контур 216
СТРОЕНИЕ АТОМА И АТОМНОГО ЯДРА.ИСПОЛЬЗОВАНИЕ ЭНЕРГИИ АТОМНЫХ ЯДЕР
65. Радиоактивные превращения атомных ядер. Строение и состав атомного ядра. Энергия общения. Дефект массы 219
Ответы 223
Таблицы физических величин 237
Литература 269

Обновлено: 13.07.2020

103583

Если вы заметили ошибку, выделите фрагмент текста и нажмите Ctrl + Enter

ГДЗ по физике 7 дидактических материалов Приони.Тема: Структура материи

Результат поиска:

  1. Физика . 7 класс Бордовый

    Марон А.Е., Марон Е.А.

    alleng.org.
  2. Физика . 7 класс . Самостоятельная и контрольная работа …

    7 класс. Самостоятельная и контрольная работа — Марон А.Е., Марон Э.А. Скачать в pdf.

    7 класс ». Пособие включает в себя самостоятельную работу в двух вариантах по каждому абзацу, тематическую тестовую работу и итоговое тестирование в четырех вариантах.

    11класов.ру.
  3. Бордовый , Бордовый : Физика . 7 класс . Дидактические материалы …

    Серия: Физика. Пособие включает обучающие задания, тесты на самоконтроль, самостоятельную работу, тестовую работу и примеры решения типовых задач.

    99101.DownLoad.
  4. Физика . 7 класс . Дидактические материалы. Бордовый A.E., Бордовый

    Образовательные ресурсы Интернета — Физика.Образовательные ресурсы Интернета — физика. Главная страница (содержание).

    Марон А.Е., Марон Е.А.

    alleng.org.
  5. Бордовый Самостоятельная и контрольная работа 7 класс физика … newgdz.com.
  6. ГДЗ П. Физика по 7 класс Maroon : Решебник с ответами …

    Марон А.Е. Физика 7 класс Дидактические материалы и GDZ для него будут одним из лучших преимуществ, которые помогут студенту быстрее и эффективнее пройти все темы, предлагаемые школьная программа.На начало изучения предмета осталось много понятий и формул …

    gDZGO.ru.
  7. Бордовый , Бордовый : Физика . 7 класс . Дидактические материалы …

    Вертикаль. GEF »и нажмите Download: Download« Марон, Марон: Физика. 7-й класс . Дидактические материалы к учебнику А.В. Пририцин. Вертикальный. ГЭФ «В.ПДФ.

    книги-Archive-2019.info.
  8. Физика 7 класс .

    Физика 7 класс. Марон А.Е.

    Пособие содержит набор вспомогательных рефератов и многоуровневых заданий, составленных в соответствии с действующим учебником физики и с новым стандартом обучения.

    Размер: 4,7 МБ. Скачать: Drive.Google.

    alleng.org.
  9. Бордовый , Бордовый , Позова: Физика . 7 класс . Сборник вопросов …

    Скачать полную версию книги «Марон, Марон, Позова: Физика.

    Марон, Марон: Физика. 7-й класс . Самостоятельная и контрольная работа к учебнику А. В. Прыскина.

    free-knizki.info.
  10. Физика . 7 класс . Самостоятельная и тестовая работа — Бордовый

    Сборник самостоятельных и контрольных работ 7 класса Марон, Марон по физике предназначен для организации текущего — тематического контроля для учащихся по учебнику Приракина «Физика. 7 класс». Включает двойную независимую работу для каждого абзаца…

    skachaj24.ru.
  11. Maroon A.E. — Скачать Электронные книги бесплатно

    Maron A.E. Физика: 7 класс.

    www.chitalkino.ru.
  12. ГДЗ: Физика 7 класс Бордовый — Дидактические материалы

    Физика 7 класс. Тип: Дидактические материалы. Авторы: Марон. Издатель: Drop. Для ребят с гуманитарным складом ума изучение формул и работа над решением задач становится серьезным испытанием. Да и любителям точных наук изучение физики не всегда легко…

    resphator.org.
  13. GDZ Физика 7 класс Бордовый — Дидактические материалы «Капля»

    Физика 7 класс. Дидактические материалы. Марон. Дрофа. В седьмом классе школьники начинают изучать физику, а это довольно сложный предмет.

    GDZ.LTD.
  14. ГДЗ П. физика 7 класс Дидактические материалы Бордовый

    Решебник по физике для 7 класса Авторы Издательство Марона Капля.

    Одним из эффективных и доступных, обрабатывающих большой блок тем и разделов считается сборник дидактических материалов по физике для 7 класса, составитель которого Маррон А.Е. В мануале разобрал …

    www.euroki. орг.
  15. Физика . Сборник вопросов и заданий. 7 -9 классы Бордовый

    7-9 классы — Марон А.Е., Марон Е.А., Позоваский С.В. Скачать в pdf.

    11класов.ру.
  16. Физика Сборник вопросов и заданий 7 -9 класс Бордовый Бордовый

    А. Э. Марон, Э. А. Марон, С. В. Позова Сборник вопросов и заданий к учебникам А. В. Прыскин, Е. М. Гуттик Физика Ороффа А. Э. Марон, Э. А

    Сборник вопросов и заданий. 7-9 кл. : учеба, пособие на общее образование. Учреждения / А. Э. Марон, Э. А. Марон, С. В. Позова.

    uchebnik-skachatj-besplatno.com.
  17. Физика . 7 класс . Дидактические материалы / Maroon A.E., Maroon

    В предлагаемом наборе 7 классных дидактических материалов авторов Марона по физике содержится более 1000 различных заданий

    Скачать учебники, учебно-методические пособия в электронном виде по гуманитарным, естественным и точным наукам для всех, кто изучает …

    skachaj24.ru.
  18. Физика . 9 класс . Самостоятельная и тестовая работа. Бордовый

    Марон А.Е., Марон Е.А.

    9 класс. Пособие включает самостоятельную работу в двух вариантах к каждому абзацу, тематические тесты по каждому разделу 9 класса урока физики в четырех вариантах и ​​два итоговых теста — по курсу физики…

    alleng.org.
  19. Maroon Поддерживаемые рефераты и многоуровневые задачи 7 класс … newgdz.com.
  20. Физика . 7 класс . Дидактические материалы — Maroon A.E., Maroon

    Скачать бесплатно без регистрации по прямой ссылке Книга Физика. 7-й класс . Дидактические материалы.

    Дидактические материалы — Марон А.Е., Марон Е.А. — 2013 Поиск книг на Math-Solution.ru.

    www.math-solution.ru.
  21. Поддержка рефератов и многоуровневых задач. Физика 7 класс

    Физика 7 класс — Марон А.Е. Скачать в pdf. Пособие содержит набор вспомогательных рефератов и многоуровневых заданий, составленных в соответствии с действующим учебником физики и новым стандартом обучения.

    11класов.ру.
  22. Решебник по физика 7 класс Бордовый — Учебные задания…

    В этом разделе подробно описаны дидактические материалы GDZ по физике для седьмого класса , что составило бордовых A.E. Maroon E.A. Среди обучающих заданий вы найдете такие темы как: «Давление жидкостей и газов», «Закон Паскаля» …

    reshimuroki.ru.
  23. Купить дидактический материал физика 7 класс , авт. Бордовый

    Пособие включает обучающие задания, тесты на самоконтроль, самостоятельную работу, контрольную работу и примеры решения типовых задач.Всего предлагаемый комплект дидактических материалов содержит более 1000 заданий и задач по следующим …

    rosuchebnik.ru.
  24. GDZ Физика 7 класс Maroon — Дидактические материалы

    GDZ по физике 7 класс Марон позволит учащимся не допускать ошибок во время проверок учителем.

    Дополнительно решебник к учебнику «Физика. Дидактические материалы 7 класс» Марон отпустит по всем непонятным темам.

    ГДЗ.Чат.
  25. Поддерживаемые тезисы I.

    А. Э. Марон Э. А. Марон. Сопровождение рефератов и многоуровневых задач. К учебнику для общеобразовательных. учреждения А. В. Прыскина «Физика. 7 класс». Санкт-Петербург 2009. УДК 373.167.1: 53 BBK22.3 M28. А.Е. Марон, доктор пед. Наук, профессор.

    school6-topki.ucoz.ru.
  26. Физика 7 Органы управления Бордовый | ТЕСТОВЫЕ ДОКУМЕНТЫ

М .: 201. 6. — 9 6 с.

Данное пособие предназначено для организации текущего и тематического контроля на занятиях, изучающих физику по учебнику А.В. Прыскин «Физика. 7 класс». Пособие включает в себя самостоятельную работу в двух версиях к каждому абзацу, тематическую тестовую работу и итоговое тестирование в четырех версиях. Качественные, расчетные и графические задания, приведенные в пособии, позволяют проверить уровень сформированности понятийного аппарата, умение применять законы физики в типовых ситуациях и организовать отражение тренировочной деятельности на уроке.

Формат: PDF.

Размер: 1 1.5 МБ

Скачать: яндекс.диск. ; РГОСТ.

СОДЕРЖАНИЕ
Предисловие 3.
Введение 4.
Самостоятельная работа
СР-1. Что изучает физика 4
Вариант 1 4.
Вариант 2 4.
CP-2. Некоторые физические термины 5
Вариант 1 5.
Вариант 2 5.
CP-3. Наблюдения и эксперименты, 6
Вариант 1 6.
Вариант 2 6.
СР-4. Физические величины. Измерение физических величин 7
Вариант 1 7.
Вариант 2 7.
CP-5. Погрешность и погрешность измерения 8
Вариант 1 8.
Вариант 2 8.
CP-6. Физика и технологии 9
Вариант 1 9.
Вариант 2 9.
Глава 1. Исходные сведения о строении вещества
Самостоятельная работа
СР-7. Состав вещества 10.
Вариант 1 10.
Вариант 2 10
СР-8. Молекулы 11.
Вариант 1 11.
Вариант 2 11.
СР-9. Броуновское движение 12.
Вариант 1 12.
Вариант 2 12.
СР-10. Распространение в газах, жидкостях и твердых телах 13
Вариант 1 13.
Вариант 2 13.
CP-11. Взаимное притяжение и отталкивание молекул 14
Вариант 1 14.
Вариант 2 14.
СР-12. Агрегированные состояния вещества 15
Вариант 1 15.
Вариант 2 15.
CP-13. Различие в молекулярной структуре твердых тел, жидкостей и газов 16
Вариант 1 16.
Вариант 2 16.
Номер экспертизы 1 17
Вариант 1 17.
Вариант 2 17.
Вариант 3 17.
Вариант 4 17.
Глава 2. Взаимодействие Тел.
Самостоятельная работа
СР-14. Механическое движение 18.
Вариант 1 18.
Вариант 2 18.
СР-15. Равномерное и неравномерное движение 19
Вариант 1 19.
Вариант 2 19.
СР-16. Скорость. Единицы скорости 20.
Вариант 1 20.
Вариант 2 20.
СР-17. Расчет пути и времени движения 21
Вариант 1 21.
Вариант 2 21.
СР-18. Инерция 22.
Вариант 1 22.
Вариант 2 22.
СР-19. Телефонное взаимодействие 23.
Вариант 1 23.
Вариант 2 23.
СР-20. Масса тела. Единицы массы 24.
Вариант 1 24.
Вариант 2 24.
СР-21. Измерение веса тела на весах 25
Вариант 1 25.
Вариант 2 25.
СР-22. Плотность вещества 26.
Вариант 1 26.
Вариант 2 26.
СР-23. Расчет массы и объема кузова по его плотности 27
Вариант 1 27.
Вариант 2 27.
СР-24. Прочность 28.
Вариант 1 28.
Вариант 2 28.
СР-25. Феномен. Сила тяжести 29.
Вариант 1 29.
Вариант 2 29.
CP-26. Сила эластичности. Закон горького 30.
Вариант 1 30.
Вариант 2 30.
CP-27. Масса корпуса 31.
Вариант 1 31.
Вариант 2 31.
СР-28. Единицы мощности. Связь между тяжестью и массой тела 32
Вариант 1 32.
Вариант 2 32.
CP-29. Гравитация на других планетах. Физические характеристики планет 33
Вариант 1 33.
Вариант 2 33.
СР-30. Динамометр 34.
Вариант 1 34.
Вариант 2 35.
CP-31. Сложение двух сил направлено одной прямой линией.
Телевизионные силы 36.
Вариант 1 36.
Вариант 2 36.
CP-32. Сила трения 37.
Вариант 1 37.
Вариант 2 37.
СР-33. Страх мира 38.
Вариант 1 38.
Вариант 2 38.
CP-34 Трение по характеру и технике 39
Вариант 1 39.
Вариант 2 39.
Номер экспертизы 2 40
Вариант 1 40.
Вариант 2 40.
Вариант 3 40.
Вариант 4 40.
Глава 3. Давление твердых тел, жидкостей и газов
Самостоятельная работа
СР-35. Давление. Единицы давления 41.
Вариант 1 41.
Вариант 2 41.
CP-36. Способы снижения и повышения давления 42
Вариант 1 42.
Вариант 2 42.
СР-37. Давление газа 43.
Вариант 1 43.
Вариант 2 43.
CP-38. Передача давления с жидкостями и газами. Закон Паскаля 44.
Вариант 1 44.
Вариант 2 44.
СР-39. Давление в жидкости и газе 45
Вариант 1 45.
Вариант 2 45.
CP-40. Расчет давления жидкости на дно и стенку сосуда 46
Вариант 1 46.
Вариант 2 46.
CP-41. Сообщающие суда 47.
Вариант 1 47.
Вариант 2 48.
CP-42. Воздух воздух. Атмосферное давление 49.
Вариант 1 49.
Вариант 2 49.
СР-43. Почему есть воздушная оболочка земли 50
Вариант 1 50.
Вариант 2 50.
СР-44. Измерение атмосферного давления.Опыт Торричелли 51.
Вариант 1 51.
Вариант 2 51.
CP-45. Барометр-анероид 52
Вариант 1 52.
Вариант 2 52.
CP-46 Атмосферное давление на разной высоте 53
Вариант 1 53.
Вариант 2 53.
CP-47. Манометры 54.
Вариант 1 54.
Вариант 2 55.
CP-48. Поршневой насос жидкости 56
Вариант 1 56.
Вариант 2, 56
CP-49. Гидравлический пресс 57.
Вариант 1 57.
Вариант 2 57.
СР-50. Воздействие жидкости и газа на погруженный в них кузов 58
Вариант 1 58.
Вариант 2 58.
CP-51. Архимедова сила 59.
Вариант 1 59.
Вариант 2 59.
CP-52. Плавательные тела 60.
Вариант 1 60.
Вариант 2 60.
CP-53. Доставка 61
Вариант 1 61.
Вариант 2 61.
CP-54. Самолет 62.
Вариант 1 62.
Вариант 2 62.
Номер экспертизы 3 63
Вариант 1 63.
Вариант 2 63.
Вариант 3 63.
Вариант 4 64.
Глава 4. Работа и мощность. ЭНЕРГИЯ
Самостоятельная работа
СР-55.Механическая работа. Единицы работы 65.
Вариант 1 65.
Вариант 2 65.
СР-56. Мощность. Блоки питания 66.
Вариант 1 66.
Вариант 2 66.
СР-57. Простые механизмы 67.
Вариант 1 67.
Вариант 2 68.
СР-58. Рычаги. Равновесные силы на рычаге 69
Вариант 1 69.
Вариант 2 69.
СР-59. Момент силы 70.
Вариант 1 70.
Вариант 2 70.
CP-60. Рычаги в технике бытового характера 71
Вариант 1 71.
Вариант 2 72.
СР-61.Правила применения Рычаг равновесия для блокировки 73
Вариант 1 73.
Вариант 2 73.
CP-62. Равенство работы при использовании простых механизмов.
Механика золотого правила 75
Вариант 1 75.
Вариант 2 75.
CP-63. ЦЕНТР НАПРАВЛЕНИЯ КУЗОВА 76
Вариант 1 76.
Вариант 2 76.
СР-64. Условия равновесия Тел. 77
Вариант 1 77.
Вариант 2 77.
CP-65. КПД механизма 78
Вариант 1 78.
Вариант 2 78.
СР-66. Энергия 79.
Вариант 1 79.
Вариант 2 79.
CP-67. Потенциальная и кинетическая энергия 80
Вариант 1 80.
Вариант 2 80.
CP-68. Преобразование одного вида механической энергии в другой 81
Вариант 1 81.
Вариант 2 81.
Номер исследования 4 82
Вариант 1 82.
Вариант 2 82.
Вариант 3 82.
Вариант 4 83.
Исследование номер 5 (окончательный) 84
Вариант 1 84.
Вариант 2 84.
Вариант 3 85.
Вариант 4 86.
Ответы 88.

Пособие включает обучающие задания, тесты на самоконтроль, самостоятельную работу, контрольную работу и примеры решения типовых задач. Всего предлагаемый комплект дидактических материалов содержит более 1000 заданий и задач по темам: «Исходные сведения о строении вещества», «взаимодействие тел», «давление твердых тел, жидкостей и газов» и «работа. и мощность. Энергия ».
Пособие адресовано учителям и учащимся общеобразовательных школ и может быть использовано при работе с различными учебниками, в которых рассмотрены актуальные темы.

Примеры.
Какое из двух тел движется с большей скоростью: 10 м за 10 м или за 4 с — 16 м? Какое из двух тел будет больше через 0,5 ч: движущееся со скоростью 36 км / ч или 12 м / с?

Трактор проехал 500 м за время равное 4 минутам, а следующие 10 минут — 2 км. Определите среднюю скорость трактора за все время движения.

Дистанцию ​​между двумя населенными пунктами мотоциклист преодолел за 30 минут, двигаясь при этом со скоростью 10 м / с.Сколько времени займет обратный путь, если он будет двигаться со скоростью 15 м / с?

Трамвай первые 50 м движется со скоростью 5 м / с, а следующие 500 м — со скоростью 10 м / с. Определите среднюю скорость трамвая на всем пути.

Во сколько пассажир, сидящий у окна поезда, движущегося со скоростью 54 км / ч, увидит проезжающий мимо него встречный поезд, скорость которого составляет 72 км / ч, если его длина составляет 150 м?

Содержание
Предисловие 3.
Учебные задания
Введение 5.
ТК-1. Измерение физических величин. Точность и погрешность измерений 5
Исходные сведения о строении вещества 6
ТЗ-2. Состав вещества 6.
Тел. 8.
ТЗ-3. Механизм 8.
ТК-4. Инерция. Взаимодействие тел. Масса корпуса 13.
ТК-5. Плотность вещества 15.
ТК-6. Феномен. Сила тяжести 17.
ТК-7. Сила эластичности. Масса корпуса 19.
ТК-8. Сила трения 20.
ТК-9. Графическое изображение сил. Сложение 21.
ТЗ-10. Силы в природе. Оценочные задания 21.
Давление твердых тел, жидкостей и газов 23
ТК-11. Давление твердых 23
TZ-12. Жидкости и газы под давлением. Закон Паскаля 24.
TK-13. Давление жидкости 25.
ТК-14. Сообщающие суда 26.
ТЗ-15. Атмосферное давление 27.
ТЗ-16. Давление в жидкостях и газах. Примерное задание 28.
ТЗ-17. Архимедова власть. Парусный тел. 30.
Работа и мощность. Энергия.Простые механизмы 32.
ТК-18. Механические работы 32.
ТЗ-19. Мощность 32.
ТЗ-20. Энергетика 33.
ТК-21. Простые механизмы. Механизмы КПД 34.
Испытания на самоконтроль
ТС-1. Состав вещества 38.
ТС-2. Механизм 40.
ТС-3. Инерция. Взаимодействие тел. Масса тела. Плотность вещества 44.
ТС-4. Силы в природе 47
ТС-5. Давление твердого тела 50
ТС-6. Давление в жидкостях и газах 53
ТС-7. Архимедова власть.Парусный спорт Тел. 56.
TS-8. Механическая работа и мощность 60
ТС-9. Энергетика 63.
ТС-10. Простые механизмы. КПД простых механизмов 64
Самостоятельная работа
СР-1. Строительный материал 68.
СР-2. Механизм 69.
СР-3. Инерция. Взаимодействие тел. Масса тела 72.
СР-4. Плотность вещества 74.
СР-5. Силы в природе 76
CP-6. Давление твердого вещества 79
CP-7. Давление в жидкостях и газах 82
СР-8. Архимедова власть. Плавательный Тел 84.
СР-9. Механическая работа. Мощность 86.
СР-10. Энергия 88.
CP-11. Простые механизмы. КПД простых механизмов 89
Контрольные работы
КР-1. Механическое движение. Плотность вещества 92.
КР-2. Давление твердых тел, жидкостей и газов 96
КР-3. Архимедова сила 100.
КР-4. Механическая работа и мощность. Простые механизмы 104.
Примеры решения типовых задач
Механизм механический 108.
Взаимодействие тел. Масса тела.Плотность вещества 109.
Масса тела. Давление твердого тела 110
Давление жидкостей и газов 111
Атмосферное давление. Архимедова сила 112.
Механическая работа и сила 114
Простые механизмы. КПД механизмов 115.
Ответы
Учебные задания 117.
Тесты на самоконтроль 117
Самостоятельная работа 119.
Экзамен 120.
Список литературы 122.


Электронную книгу бесплатно скачать в удобном формате, посмотреть и прочитать :
Скачать книгу по физике, 7 класс, Учебное пособие, Марон А.Э., Марон Э.А., 2013 — FilesKachat.com, Быстрая и бесплатная загрузка.

Скачать файл №1 — PDF
Скачать файл №2 — DJVU
Ниже вы можете купить эту книгу по лучшей цене со скидкой с доставкой по России.

Онлайн-тренинг по физике. Как начать изучение физики с абсолютного нуля? (В школе ничему не научился)? С физикой проще и интереснее изучать физику с Виртуальной академией

Эта книга позволит читателю легко освоить основы школьного курса физики.Автор поможет разобраться в сути основных законов и явлений физики, не углубляясь в сложные теоретические выкладки. В книге собраны основные сведения из основных областей физики: кинематики, механики, термодинамики, электромагнетизма и оптики. Все пояснения сопровождаются простыми примерами, которые не претендуют на полное описание физических процессов, но позволяют быстро понять их суть.

Наблюдаем за движущимися объектами.
Некоторые из самых фундаментальных вопросов об устройстве мира связаны с движением объектов. Будет ли замедляться катящийся к вам огромный камень? Насколько быстро нужно двигаться, чтобы не столкнуться с ним? (Погодите, теперь я рассчитываю на калькулятор …) Движение было одной из первых тем исследований, которыми физики давно занимаются, пытаясь получить убедительные ответы на свои вопросы.

В части I этой книги исследуется движение объектов, от бильярдных шаров до железнодорожных вагонов.Движение — фундаментальное явление в нашей жизни и одно из тех явлений, о которых большинство людей знает толк. Достаточно нажать на педаль газа, и машина тронется.

Но не все так просто. Описание принципов движения — это первый шаг к пониманию физики, которая проявляется в наблюдениях и измерениях, а также в создании мысленных и математических моделей, основанных на этих наблюдениях и измерениях. Этот процесс незнаком большинству людей, и книга для этого предназначена.

Казалось бы, простой процесс обучения движению — это начало начала. Если вы присмотритесь, вы заметите, что реальное движение постоянно меняется. Наблюдайте, как мотоцикл тормозит на светофоре, лист, падающий на землю и продолжающий двигаться под действием ветра, при невероятном движении бильярдных шаров после хитрого удара мастера.

Содержание
Введение
Часть I. Мир в движении
Глава 1.Как понять наш мир с помощью физики
Глава 2. Понимание основ физики
Глава 3. Удовлетворение потребности в скорости
Глава 4. Идем по знакам
Часть II. Да пребудут с нами силы физики
Глава 5. Стремление к силе: мощь
Глава 6. Использование ремней безопасности: наклонные плоскости и трение
Глава 7. Движение по орбитам
Часть III. Превращение работы в энергию и наоборот
Глава 8.Выполнение работы
Глава 9. Движущиеся объекты: импульс и импульс
Глава 10. Вращающиеся объекты: момент силы
Глава 11. Вращающиеся объекты: момент инерции
Глава 12. Сжимающие пружины: простое гармоническое движение
Часть IV. Формулирование законов термодинамики
Глава 13. Неожиданное объяснение тепла с помощью термодинамики
Глава 14. Мы передаем тепловую энергию в твердых телах и газах
Глава 15. Тепловая энергия и работа: истоки термодинамики
Часть V.Электрификация и намагничивание
Глава 16. Электрификация: исследование статического электричества
Глава 17. Мы летим за электронами по проводам
Глава 18. Намагниченность: притягиваемся и отталкиваемся
Глава 19. Укрощение колебаний тока и напряжения
Глава 20. Немного свет на зеркала и линзы
Часть VI. Великолепные десятки
Глава 21. Десять удивительных гипотез теории относительности
Глава 22. Десять безумных физических идей Глоссарий
Предметный указатель.


Скачать бесплатно электронную книгу в удобном формате, смотреть и читать:
Скачать книгу Физика для чайников, Хольцнер С., 2012 — fileskachat.com, быстро и бесплатно скачать.

Возможны несколько вариантов в зависимости от вашей цели, свободного времени и уровня математической подготовки.

Вариант 1

Цель «для себя», временные рамки не ограничены, математика тоже практически с нуля.

Выберите ряд учебников, которые более интересны, например, трехтомник Ландсберга, и изучите его, делая заметки в тетради.Затем полистайте учебники Г.Я. Мякишева и Б.Буховцева для 10-11 классов аналогично. Подкрепите полученные знания — прочтите справочник для 7-11 классов А.Ф. Кабардиной.

Если учебные пособия Г.С. Ландсберга вам не подошли, и они предназначены для тех, кто изучает физику с нуля, возьмите линейку учебников для 7-9 классов А.В. Перышкин и Е.М.Гутник. Нечего стыдиться, что это для маленьких детей — порой пятиклассники без подготовки «плывут» в Перышкине за 7 класс уже с десятой страницы.

Как сделать

Обязательно отвечайте на вопросы и решайте задачи после абзацев.

В конце тетради составьте для себя справочник основных понятий и формул.

Обязательно найдите на YouTube видео с физическими переживаниями, которые есть в учебнике. Просмотрите и обведите их по схеме: что вы видели — что наблюдали — почему? Рекомендую ресурс GetAClass — там систематизированы все эксперименты и теория по ним.

Сразу завести отдельную записную книжку для решения проблем. Начнем с задачника В.И. Лукашик, Е.В. Ивановой для 7-9 классов и решить половину заданий из нее. Затем решите на 70% задачник А.П. Рымкевича или, как вариант — «Сборник вопросов и задач по физике» для 10-11 классов Г. Н. и А. П. Степановых.

Попробуйте определиться самостоятельно, загляните в рещебник на крайний случай. Если вы столкнулись с трудностью, ищите аналог проблемы с парсингом.Для этого нужно иметь под рукой 3-4 бумажных книжки, где подробно обсуждаются решения физических задач. Например, «Проблемы физики с анализом их решения» Н.Е. Савченко или книги И. Л. Касаткиной.

Если вам все будет понятно, и душа будет просить сложных вещей — возьмите многотомную книгу Г.Я. Мякишева, А.З. Синякова за профильные занятия и решение всех упражнений.

Приглашаем всех изучать физику

Вариант 2

Цель — экзамен или другой экзамен, срок два года, математика с нуля.

Справочник школьников О. Ф. Кабардиной и «Сборник задач по физике» для 10-11 классов О. И. Громцева О. И. («заточена» к ЕГЭ). Если экзамен не является ЕГЭ, лучше сдавать тестовые тетради В. И. Лукашика и А. П. Рымкевича или «Сборник вопросов и задач по физике» для 10-11 классов Г. Н. Степановой, А. П. Степанова. Смело обращайтесь к учебникам А.В. Перышкина и Э.М.Гутника для 7-9 классов, а точнее, просмотрите их тоже.

Настойчивые и трудолюбивые могут полностью пройти по книге «Физика. Полный курс школы» В. А. Орлов, Г. Г. Никифоров, А. А. Фадеева и другие. В этом пособии есть все необходимое: теория, практика, задания.

Как это сделать

Система такая же, как и в первой версии:

  • вести записные книжки для лекций и решения проблем,
  • самостоятельно делать заметки и решать проблемы в записной книжке,
  • просматривать и анализировать опыты, например, на GetAClass.
  • Если вы хотите максимально эффективно подготовиться к ЕГЭ или ЕГЭ в оставшееся время,
    Вариант 3

Цель — ЕГЭ, семестр — 1 год, математика на хорошем уровне.

Если математика нормальная, можно не обращаться к учебникам 7-9 классов, а сразу брать 10-11 классы и справочник для школьников О. Ф. Кабардин. В кабардинском пособии есть темы, которых нет в учебниках 10-11 классов.При этом рекомендую посмотреть видео с физическими экспериментами и проанализировать их по схеме.

Вариант 4

Цель — ЕГЭ, семестр — 1 год, математика — на ноль.

Подготовиться к экзамену за год без базы по математике нереально. Если только вы не будете выполнять все пункты из варианта №2 каждый день в течение 2 часов.

Учителя и наставники Foxford Online School помогут добиться максимального результата за оставшееся время.

М .: 2010. — 752с. М .: 1981. — Т. 1 — 336с., Т. 2 — 288с.

Книга известного физика из США Дж. Орира — один из самых успешных вводных курсов по физике в мировой литературе, охватывающий диапазон от физики как школьного предмета до доступного описания ее последних достижений. Эта книга заняла почетное место на книжной полке для нескольких поколений российских физиков, и к этому изданию книга была существенно дополнена и модернизирована.Автор книги, ученик выдающегося физика ХХ века, лауреата Нобелевской премии Э. Ферми, долгие годы преподавал свой курс студентам Корнельского университета. Этот курс может служить полезным практическим введением в известные российские лекции Фейнмана по физике и курс физики в Беркли. По уровню и содержанию книга Ориры уже доступна старшеклассникам, но также может быть интересна школьникам, аспирантам, преподавателям, а также всем тем, кто желает не только систематизировать и пополнить свои знания в области физики, но и научиться успешно решать широкий класс физических задач.

Формат: pdf (2010, 752с.)

Размер: 56 Мб

Часы, скачать: drive.google

Примечание. Ниже представлено цветное сканирование.

Том 1.

Формат: djvu (1981, 336 с.)

Размер: 5,6 МБ

Часы, скачать: drive.google

Том 2.

Формат: djvu (1981, 288 с.)

Размер: 5,3 МБ

Часы, скачать: drive.google

СОДЕРЖАНИЕ
Предисловие редактора русского издания 13
Предисловие 15
1. ВВЕДЕНИЕ 19
§ 1. Что такое физика? девятнадцать
§ 2. Единицы измерения 21
§ 3. Анализ размеров 24
§ 4. Точность в физике 26
§ 5. Роль математики в физике 28
§ 6. Наука и общество 30
Применение.Правильные ответы без типичных ошибок 31
Упражнения 31
Задачи 32
2. ОДНОМЕРНОЕ ДВИЖЕНИЕ 34
§ 1. Скорость 34
§ 2. Средняя скорость 36
§ 3. Ускорение 37
§ 4. Равномерное ускорение движение 39
Основные выводы 43
Упражнения 43
Задачи 44
3. ДВУХМЕРНОЕ ДВИЖЕНИЕ 46
§ 1. Траектории свободного падения 46
§ 2. Векторы 47
§ 3. Движение снаряда 52
§ 4. Равномерное движение по кругу 24
§ 5.Искусственные спутники Земли 55
Основные выводы 58
Упражнения 58
Задачи 59
4. ДИНАМИКА 61
§ 1. Введение 61
§ 2. Определения основных понятий 62
§ 3. Законы Ньютона 63
§ 4. Единицы силы и массы 66
§ 5. Контактные силы (силы реакции и трения) 67
§ 6. Решение задач 70
§ 7. Машина Этвуда 73
§ 8. Конический маятник 74
§ 9. Закон сохранения количества движения 75
Основные выводы 77
Упражнения 78
Задачи 79
5.ГРАВИТАЦИЯ 82
§ 1. Закон всемирного тяготения 82
§ 2. Опыт Кавендиша 85
§ 3. Законы Кеплера для движения планет 86
§ 4. Вес 88
§ 5. Принцип эквивалентности 91
§ 6. Гравитационное поле внутри сферы 92
Основные выводы 93
Упражнения 94
Задачи 95
6. РАБОТА И ЭНЕРГИЯ 98
§ 1. Введение 98
§ 2. Работа 98
§ 3. Мощность 100
§ 4. Точечное произведение 101
§ 5. Кинетическая энергия 103
§ 6.Потенциальная энергия 105
§ 7. Гравитационная потенциальная энергия 107
§ 8. Потенциальная энергия пружины 108
Основные выводы 109
Упражнения 109
Задания 111
7. ЗАКОН СОХРАНЕНИЯ ЭНЕРГИИ ОТ
§ 1. Сохранение механической энергии энергия 114
§ 2. Столкновения 117
§ 3. Сохранение гравитационной энергии 120
§ 4. Диаграммы потенциальной энергии 122
§ 5. Сохранение полной энергии 123
§ 6. Энергия в биологии 126
§ 7.Энергия и автомобиль 128
Основные выводы 131
Применение. Закон сохранения энергии для системы из N частиц 131
Упражнения 132
Задачи 132
8. ОТНОСИТЕЛЬНАЯ КИНЕМАТИКА 136
§ 1. Введение 136
§ 2. Постоянство скорости света 137
§ 3. Замедление времени 142
§ 4. Преобразования Лоренца 145
§ 5. Одновременность 148
§ 6. Оптический эффект Доплера 149
§ 7. Парадокс близнецов 151
Основные выводы 154
Упражнения 154
Задачи 155
9.РЕЛЯТИВИСТСКАЯ ДИНАМИКА 159
§ 1. Релятивистское сложение скоростей 159
§ 2. Определение релятивистского импульса 161
§ 3. Закон сохранения количества движения и энергии 162
§ 4. Эквивалентность массы и энергии 164
§ 5. Кинетическая энергия 166
§ 6. Масса и сила 167
§ 7. Общая теория относительности 168
Основные выводы 170
Применение. Преобразование энергии и импульса 170
Упражнения 171
Примеры 172
10. ВРАЩАТЕЛЬНОЕ ДВИЖЕНИЕ 175
§ 1.Кинематика вращательного движения 175
§ 2. Векторное произведение 176
§ 3. Момент импульса 177
§ 4. Динамика вращательного движения 179
§ 5. Центр масс 182
§ 6. Жесткие тела и момент инерции 184
§ 7. Статика 187
§ 8. Маховики 189
Основные выводы 191
Упражнения 191
Задачи 192
11. Колебательные движения 196
§ 1. Гармоническая сила 196
§ 2. Период колебаний 198
§ 3. Маятник 200
§ 4. Энергия простого гармонического движения 202
§ 5.Малые колебания 203
§ 6. Интенсивность звука 206
Основные выводы 206
Упражнения 208
Примеры 209
12. КИНЕТИЧЕСКАЯ ТЕОРИЯ 213
§ 1. Давление и гидростатика 213
§ 2. Уравнение состояния идеального газа 217
§ 3. Температура 219
§ 4. Равномерное распределение энергии 222
§ пять. Кинетическая теория тепла 224
Основные выводы 226
Упражнения 226
Примеры 228
13. ТЕРМОДИНАМИКА 230
§ 1. Первое начало термодинамики 230
§ 2.Гипотеза Авогадро 231
§ 3. Удельная теплоемкость 232
§ 4. Изотермическое расширение 235
§ 5. Адиабатическое расширение 236
§ 6. Бензиновый двигатель 238
Основные выводы 240
Упражнения 241
Задачи 241
14. ВТОРОЙ ЗАКОН ТЕРМОДИНАМИКИ 244
§ 1. Машина Карно 244
§ 2. Тепловое загрязнение окружающей среды 246
§ 3. Холодильники и тепловые насосы 247
§ 4. Второй закон термодинамики 249
§ 5. Энтропия 252
§ 6. Обращение времени 256
Основные выводы 259
Упражнения 259
Случаи 260
15.ЭЛЕКТРОСТАТИЧЕСКАЯ СИЛА 262
§ 1. Электрический заряд 262
§ 2. Закон Кулона 263
§ 3. Электрическое поле 266
§ 4. Линии электропередач 268
§ 5. Теорема Гаусса 270
Основные выводы 275
Упражнения 275
Случаи 276
16. ЭЛЕКТРОСТАТИКА 279
§ 1. Сферическое распределение заряда 279
§ 2. Линейное распределение заряда 282
§ 3. Плоское распределение 283
§ 4. Электрический потенциал 286
§ 5. Электрическая мощность 291
§ 6.Диэлектрики 294
Основные выводы 296
Упражнения 297
Примеры 299
17. ЭЛЕКТРИЧЕСКИЙ ТОК И МАГНИТНАЯ СИЛА 302
§ 1. Электрический ток 302
§ 2. Закон Ома 303
§ 3. Цепи постоянного тока 306
§ 4. Эмпирические данные о магнитной силе 310
§ 5. Вывод формулы для магнитной силы 312
§ 6. Магнитное поле 313
§ 7. Единицы измерения магнитного поля 316
§ 8. Релятивистское преобразование величин * 8 и E 318
Основные выводы 320
Приложение.Релятивистские преобразования тока и заряда 321
Практические упражнения 322
Случаи 323
18. МАГНИТНЫЕ ПОЛЯ 327
§ 1. Закон Ампера 327
§ 2. Некоторые конфигурации токов 329
§ 3. Закон Bio-Savard 333
§ 4. Магнетизм 336
§ 5. Уравнения Максвелла для постоянных токов 339
Основные выводы 339
Упражнения 340
Случаи 341
19. ЭЛЕКТРОМАГНИТНАЯ ИНДУКЦИЯ 344
§ 1. Двигатели и генераторы 344
§ 2.Закон Фарадея 346
§ 3. Закон Ленца 348
§ 4. Индуктивность 350
§ 5. Энергия магнитного поля 352
§ 6. Цепи переменного тока 355
§ 7. Цепи RC и RL 359
Основные выводы 362
Применение. Freeform Path 363
Упражнения 364
Cases 366
20. ЭЛЕКТРОМАГНИТНОЕ ИЗЛУЧЕНИЕ И ВОЛНЫ 369
§ 1. Ток смещения 369
§ 2. Уравнения Максвелла в целом 371
§ 3. Электромагнитное излучение 373
§ 4. Излучение плоской синусоидальной формы. текущий 374
§ 5.Несинусоидальный ток; Разложение Фурье 377
§ 6. Бегущие волны 379
§ 7. Передача энергии волнами 383
Основные выводы 384
Приложение. Вывод волнового уравнения 385
Упражнения 387
Примеры 387
21. ВЗАИМОДЕЙСТВИЕ ИЗЛУЧЕНИЯ С ВЕЩЕСТВОМ 390
§ 1. Энергия излучения 390
§ 2. Импульс излучения 393
§ 3. Отражение излучения от хорошего проводника 394
§ 4. Взаимодействие излучения с диэлектриком 395
§ 5.Показатель преломления 396
§ 6. Электромагнитное излучение в ионизированной среде 400
§ 7. Поле излучения точечных зарядов 401
Основные выводы 404
Приложение 1. Метод фазовых диаграмм 405
Приложение 2. Волновые пакеты и 406 Групповая скорость
Упражнения 410
Случаи 410
22. ИНТЕРФЕРЕНЦИЯ ВОЛН 414
§ 1. Стоячие волны 414
§ 2. Интерференция волн от двух точечных источников 417
§3. Интерференция волн от большого количества источников 419
§ 4.Дифракционная решетка 421
§ 5. Принцип Гюйгенса 423
§ 6. Дифракция на отдельной щели 425
§ 7. Когерентность и некогерентность 427
Основные выводы 430
Упражнение 431
Случаи 432
23. ОПТИКА 434
§ 1. Голография 434
§ 2. Поляризация света 438
§ 3. Дифракция в круглом отверстии 443
§ 4. Оптические устройства и их разрешение 444
§ 5. Дифракционное рассеяние 448
§ 6. Геометрическая оптика 451
Основные выводы 455
Применение .Закон Брюстера 455
Упражнение 456
Присваивания 457
24. ВОЛНОВАЯ ПРИРОДА ВЕЩЕСТВА 460
§ 1. Классическая и современная физика 460
§ 2. Фотоэффект 461
§ 3. Эффект Комптона 465
§ 4. Волновая корпускула дуализм 465
§ 5. Великий парадокс 466
§ 6. Дифракция электронов 470
Основные выводы 472
Практические упражнения 473
Случаи 473
25. КВАНТОВАЯ МЕХАНИКА 475
§ 1. Волновые пакеты 475
§ 2. Принцип неопределенности 477
§ 3.Частица в ящике 481
§ 4. Уравнение Шредингера 485
§ 5. Потенциальные ямки конечной глубины 486
§ 6. Гармонический осциллятор 489
Основные выводы 491
Упражнения 491
Случаи 492
26. АТОМ ВОДОРОДА 495
§ 1. Приближенная теория атома водорода 495
§ 2. Трехмерное уравнение Шредингера 496
§ 3. Строгая теория атома водорода 498
§ 4. Орбитальный угловой момент 500
§ 5. Эмиссия фотонов 504
§ 6.Вынужденное излучение 508
§ 7. Модель атома Бора 509
Основные выводы 512
Практические упражнения 513
Примеры 514
27. АТОМНАЯ ФИЗИКА 516
§ 1. Принцип исключения Паули 516
§ 2. Многоэлектронные атомы 517
§ 3. Периодическая система из 521 элемента
§ 4. Рентгеновское излучение 525
§ 5. Связывание в молекулах 526
§ 6. Гибридизация 528
Основные выводы 531
Практические упражнения 531
Кейсы 532
28. КОНДЕНСИРОВАННЫЕ СРЕДЫ 533
§ 1.Типы связи 533
§ 2. Теория свободных электронов в металлах 536
§ 3. Электропроводность 540
§ 4. Зонная теория твердого тела 544
§ 5. Физика полупроводников 550
§ 6. Сверхтекучесть 557
§ 7. Проникновение через барьер 558
Основные выводы 560
Применение. Различные приложения /? — n-переход (в радио и телевидении) 562
Упражнения 564
Кейсы 566
29. ЯДЕРНАЯ ФИЗИКА 568
§ 1. Размеры ядер 568
§ 2.Фундаментальные силы, действующие между двумя нуклонами 573
§ 3. Строение тяжелых ядер 576
§ 4. Альфа-распад 583
§ 5. Гамма- и бета-распады 586
§ 6. Деление ядер 588
§ 7. Синтез ядер 592
Ключевые выводы 596
Практические упражнения 597
Примеры 597
30. АСТРОФИЗИКА 600
§ 1. Источники энергии звезд 600
§ 2. Эволюция звезд 603
§ 3. Квантово-механическое давление вырожденного ферми-газа 605
§ 4.Белые карлики 607
§ 6. Черные дыры 609
§ 7. Нейтронные звезды 611
31. ФИЗИКА ЭЛЕМЕНТАРНЫХ ЧАСТИЦ 615
§ 1. Введение 615
§ 2. Фундаментальные частицы 620
§ 3. Фундаментальные взаимодействия 622
§ 4 .Взаимодействия между фундаментальными частицами как обмен квантами несущего поля 623
§ 5. Симметрии в мире частиц и законы сохранения 636
§ 6. Квантовая электродинамика как локальная калибровочная теория 629
§ 7. Внутренние симметрии адронов 650
§ 8.Кварковая модель адронов 636
§ 9. Цвет. Квантовая хромодинамика 641
§ 10. «Видны» ли кварки и глюоны? 650
§ 11. Слабые взаимодействия 653
§ 12. Несохранение четности 656
§ 13. Промежуточные бозоны и неперенормируемость теории 660
§ 14. Стандартная модель 662
§ 15. Новые идеи: TVO, суперсимметрия, суперструны 674
32. ГРАВИТАЦИЯ И КОСМОЛОГИЯ 678
§ 1. Введение 678
§ 2. Принцип эквивалентности 679
§ 3.Метрические теории гравитации 680
§ 4. Структура уравнений общей теории относительности. Простейшие решения 684
§ 5. Проверка принципа эквивалентности 685
§ 6. Как оценить масштаб эффектов общей теории относительности? 687
§ 7. Классические тесты общей теории относительности 688
§ 8. Основные принципы современной космологии 694
§ 9. Модель горячей Вселенной («стандартная» космологическая модель) 703
§ 10. Возраст Вселенной 705
§ одиннадцать .Критическая плотность и сценарии эволюции Фридмана 705
§ 12. Плотность вещества во Вселенной и скрытая масса 708
§ 13. Сценарий первых трех минут эволюции Вселенной 710
Раздел 14. Ближе к началу 718
§ 15. Сценарий инфляции 722
§ 16. Тайна темной материи 726
ПРИЛОЖЕНИЕ A 730
Физические константы 730
Некоторая астрономическая информация 730
ПРИЛОЖЕНИЕ B 731
Единицы измерения основных физических величин 731
Единицы измерения электрических величин 731
ПРИЛОЖЕНИЕ B 732
Геометрия 732
Тригонометрия 732
Квадратичное уравнение 732
Некоторые производные 733
Некоторые неопределенные интегралы (до произвольной постоянной) 733
Произведения векторов 733
Греческий алфавит 733
ОТВЕТЫ НА УПРАЖНЕНИЯ И ПРОБЛЕМЫ 734
ИНДЕКС 746

В настоящее время практически отсутствуют участки естественного наука или технические знания, где достижения физики в той или иной степени не использовались бы.Более того, эти достижения все больше проникают в традиционные гуманитарные науки, что находит отражение во включении дисциплины «Концепции современного естествознания» в учебные планы всех гуманитарных специальностей российских вузов.
Предлагаемая вниманию российского читателя книга Дж. Орира впервые была издана в России (точнее, в СССР) более четверти века назад, но, как и в случае с действительностью, хорошие книги, до сих пор не потерял интереса и актуальности.Секрет жизненной силы книги Ориера заключается в том, что она успешно заполняет нишу, неизменно востребованную всеми новыми поколениями читателей, в основном молодыми.
Не являясь учебником в обычном понимании этого слова и не претендуя на то, чтобы заменить его, книга Ориера предлагает довольно полное и последовательное изложение всего курса физики на совершенно элементарном уровне. Этот уровень не отягощен сложной математикой и в принципе доступен каждому любознательному и трудолюбивому ученику, а тем более ученику.
Легкий и свободный стиль изложения, не жертвующий логикой и не избегающий сложных вопросов, продуманный подбор иллюстраций, диаграмм и графиков, использование большого количества примеров и заданий, которые, как правило, носят практический характер. важность и соответствие жизненному опыту учащихся — все это делает книгу Ориера незаменимым помощником для самообразования или дополнительного чтения.
Конечно, его можно с успехом использовать как полезное дополнение к привычным учебникам и учебникам по физике, прежде всего в физико-математических классах, лицеях и колледжах.Книгу Орира можно также рекомендовать студентам высших учебных заведений, в которых физика не является основной дисциплиной.

Физика — одна из фундаментальных наук естествознания. Изучение физики в школе начинается с 7 класса и продолжается до конца школы. К этому времени школьники должны уже сформировать правильный математический аппарат, необходимый для изучения курса физики.

  • Школьная программа по физике состоит из нескольких больших разделов: механика, электродинамика, колебания и волны, оптика, квантовая физика, молекулярная физика и тепловые явления.

Школьные темы по физике

В 7 классе происходит поверхностное знакомство и введение в курс физики. Рассмотрены основные физические представления, изучается структура веществ, а также сила давления, с которой одни вещества действуют на другие. Кроме того, изучаются законы Паскаля и Архимеда.

В 8 классе изучаются различные физические явления. Дается начальная информация о магнитном поле и явлениях, в которых оно возникает.Постоянное электричество и основные законы оптики. Различные агрегатные состояния вещества и процессы, происходящие при переходе вещества из одного состояния в другое.

9 класс посвящен основным законам движения тел и их взаимодействия друг с другом. Основные понятия охватываются механическими колебаниями и волнами. Тема звука и звуковых волн рассматривается отдельно. Изучены основы теории электромагнитных полей и электромагнитных волн.Кроме того, есть знакомство с элементами ядерной физики и изучение строения атома и атомного ядра.

В 10 классе начинается углубленное изучение механики (кинематики и динамики) и законов сохранения. Рассмотрены основные виды механических сил. Происходит углубленное изучение тепловых явлений, изучаются молекулярно-кинетическая теория и основные законы термодинамики. Повторены и систематизированы основы электродинамики: электростатика, законы постоянного электрического тока и электрического тока в различных средах.

11 класс посвящен изучению магнитного поля и явления электромагнитной индукции. Подробно изучаются различные виды колебаний и волн: механические и электромагнитные. Идет углубление знаний из раздела оптики. Рассмотрены элементы теории относительности и квантовой физики.

  • Ниже приведен список с 7 по 11 классы. Каждый класс содержит темы по физике, написанные нашими преподавателями. Эти материалы могут использоваться как учениками, так и их родителями, а также школьными учителями и воспитателями.

Физика 7 дидактических материалов Перышкин. Физические диктанты типа II

М .: 201 6. — 9 6 с.

Пособие предназначено для организации текущего и тематического контроля на занятиях по физике по учебнику А.В. Перышкина «Физика. 7 класс ». Пособие включает самостоятельную работу в двух вариантах по каждому абзацу, тематические тесты и итоговый тест в четырех вариантах. Приведенные в пособии качественные, вычислительные и графические задания позволяют проверить уровень сформированности понятийного аппарата, умение применять законы физики в типичных ситуациях и организовывать отражение учебной деятельности на уроке.

Формат: pdf

Размер: 1 1.5 Мб

Скачать: yandex.disk ; Rghost

СОДЕРЖАНИЕ
Предисловие 3
Введение 4
НЕЗАВИСИМЫЕ РАБОТЫ
CP-1. Что изучает физика 4
Вариант 1 4
Вариант 2 4
CP-2. Некоторые физические термины 5
Вариант 1 5
Вариант 2 5
CP-3. Наблюдения и эксперименты, 6
Вариант 1 6
Вариант 2 6
CP-4.Физические величины. Измерение физических величин 7
Вариант 1 7
Вариант 2 7
CP-5. Точность и погрешность измерения 8
Вариант 1 8
Вариант 2 8
CP-6. Физика и технологии 9
Вариант 1 9
Вариант 2 9
Глава 1. ИСХОДНАЯ ИНФОРМАЦИЯ О СТРУКТУРЕ ВЕЩЕСТВА
НЕЗАВИСИМЫЕ РАБОТЫ
СР-7. Состав вопроса 10
Вариант 1 10
Вариант 2 10
CP-8. Молекулы 11
Вариант 1 11
Вариант 2 11
CP-9.Броуновское движение 12
Вариант 1 12
Вариант 2 12
CP-10. Распространение в газах, жидкостях и твердых телах 13
Вариант 1 13
Вариант 2 13
CP-11. Взаимное притяжение и отталкивание молекул 14
Вариант 1 14
Вариант 2 14
CP-12. Агрегатное состояние 15
Вариант 1 15
Вариант 2 15
CP-13. Разница в молекулярной структуре твердых тел, жидкостей и газов 16
Вариант 1 16
Вариант 2 16
КОНТРОЛЬНАЯ РАБОТА № 1 17
Вариант 1 17
Вариант 2 17
Вариант 3 17
Вариант 4 17
Глава 2.ВЗАИМОДЕЙСТВИЕ ОРГАНОВ
НЕЗАВИСИМЫЕ РАБОТЫ
СР-14. Механический механизм 18
Вариант 1 18
Вариант 2 18
CP-15. Равномерное и неравномерное движение 19
Вариант 1 19
Вариант 2 19
CP-16. Скорость. Единицы скорости 20
Вариант 1 20
Вариант 2 20
СР-17. Расчет расстояния и времени в пути 21
Вариант 1 21
Вариант 2 21
CP-18. Инерция 22
Вариант 1 22
Вариант 2 22
SR-19. Взаимодействие тел 23
Вариант 1 23
Вариант 2 23
SR-20.Масса тела. Единицы измерения массы 24
Вариант 1 24
Вариант 2 24
CP-21. Измерение веса тела на весах 25
Вариант 1 25
Вариант 2 25
CP-22. Плотность вещества 26
Вариант 1 26
Вариант 2 26
CP-23. Расчет массы и объема тела по его плотности 27
Вариант 1 27
Вариант 2 27
СР-24. Прочность 28
Вариант 1 28
Вариант 2 28
SR-25. Явление гравитации. Гравитация 29
Вариант 1 29
Вариант 2 29
SR-26.Прочность эластичности. Закон Гука 30
Вариант 1 30
Вариант 2 30
SR-27. Масса корпуса 31
Вариант 1 31
Вариант 2 31
CP-28. Силовые единицы. Связь между гравитацией и массой тела 32
Вариант 1 32
Вариант 2 32
SR-29. Гравитация на других планетах. Физические характеристики планет 33
Вариант 1 33
Вариант 2 33
CP-30. Динамометр 34
Вариант 1 34
Вариант 2 35
SR-31. Сложение двух сил, направленных по одной прямой.
Действующие силы 36
Вариант 1 36
Вариант 2 36
CP-32. Сила трения 37
Вариант 1 37
Вариант 2 37
CP-33. Остаточное трение 38
Вариант 1 38
Вариант 2 38
SR-34 Трение по природе и технологии 39
Вариант 1 39
Вариант 2 39
КОНТРОЛЬНАЯ РАБОТА № 2 40
Вариант 1 40
Вариант 2 40
Вариант 3 40
Вариант 4 40
Глава 3. ДАВЛЕНИЕ ТВЕРДЫХ, ЖИДКОСТЕЙ И ГАЗОВ
НЕЗАВИСИМЫЕ РАБОТЫ
SR-35.Давление. Единицы давления 41
Вариант 1 41
Вариант 2 41
CP-36. Способы понижения и повышения давления 42
Вариант 1 42
Вариант 2 42
CP-37. Давление газа 43
Вариант 1 43
Вариант 2 43
CP-38. Передача давления жидкостями и газами. Закон Паскаля 44
Вариант 1 44
Вариант 2 44
CP-39. Давление в жидкости и газе 45
Вариант 1 45
Вариант 2 45
SR-40. Расчет давления жидкости на дно и стенки сосуда 46
Вариант 1 46
Вариант 2 46
CP-41.Сообщающиеся сосуды 47
Вариант 1 47
Вариант 2 48
SR-42. Воздушный вес. Атмосферное давление 49
Вариант 1 49
Вариант 2 49
SR-43. Почему существует воздушная оболочка Земли 50
Вариант 1 50
Вариант 2 50
SR-44. Измерение атмосферного давления. Опыт Торричелли 51
Вариант 1 51
Вариант 2 51
CP-45. Барометр-анероид 52
Вариант 1 52
Вариант 2 52
SR-46 Атмосферное давление на разной высоте 53
Вариант 1 53
Вариант 2 53
CP-47.Манометры 54
Вариант 1 54
Вариант 2 55
CP-48. Поршневой жидкостный насос 56
Вариант 1 56
Вариант 2, 56
CP-49. Гидравлический пресс 57
Вариант 1 57
Вариант 2 57
CP-50. Воздействие жидкости и газа на погруженное в них тело 58
Вариант 1 58
Вариант 2 58
СР-51. Архимедова сила 59
Вариант 1 59
Вариант 2 59
CP-52. Плавательные тела 60
Вариант 1 60
Вариант 2 60
CP-53. Навигационные суда 61
Вариант 1 61
Вариант 2 61
CP-54.Аэронавтика 62
Вариант 1 62
Вариант 2 62
КОНТРОЛЬНАЯ РАБОТА № 3 63
Вариант 1 63
Вариант 2 63
Вариант 3 63
Вариант 4 64
Глава 4. РАБОТА И МОЩНОСТЬ. ЭНЕРГИЯ
НЕЗАВИСИМЫЕ РАБОТЫ
CP-55. Механическая работа. Рабочие единицы 65
Вариант 1 65
Вариант 2 65
CP-56. Мощность. Блоки питания 66
Вариант 1 66
Вариант 2 66
CP-57. Простые механизмы 67
Вариант 1 67
Вариант 2 68
CP-58. Рычаги.Баланс сил на рычаге 69
Вариант 1 69
Вариант 2 69
СР-59. Момент мощности 70
Вариант 1 70
Вариант 2 70
CP-60. Рычаги в технике, жизнь на природе 71
Вариант 1 71
Вариант 2 72
CP-61. Применение правила балансировки рычага к блоку 73
Вариант 1 73
Вариант 2 73
CP-62. Равенство работы при использовании простых механизмов.
«Золотое правило» механики 75
Вариант 1 75
Вариант 2 75
SR-63. Центр тяжести корпуса 76
Вариант 1 76
Вариант 2 76
CP-64.Условия равновесия тел 77
Вариант 1 77
Вариант 2 77
CP-65. КПД механизма 78
Вариант 1 78
Вариант 2 78
СР-66. Энергия 79
Вариант 1 79
Вариант 2 79
CP-67. Потенциальная и кинетическая энергия 80
Вариант 1 80
Вариант 2 80
SR-68. Преобразование одного вида механической энергии в другой 81
Вариант 1 81
Вариант 2 81
КОНТРОЛЬНАЯ РАБОТА № 4 82
Вариант 1 82
Вариант 2 82
Вариант 3 82
Вариант 4 83
КОНТРОЛЬНАЯ РАБОТА №5 (окончательный) 84
Вариант 1 84
Вариант 2 84
Вариант 3 85
Вариант 4 86
ОТВЕТЫ 88

Пособие включает обучающие задания, тесты на самоконтроль, самостоятельную работу, контрольную работу и примеры решения типовых задач. Всего предлагаемый комплект дидактических материалов содержит более 1000 заданий и заданий по следующим темам: «Исходные сведения о строении вещества», «Взаимодействие тел», «Давление твердых тел, жидкостей и газов» и «Работа и мощность.Энергия ».
Пособие адресовано учителям и учащимся общеобразовательных школ и может быть использовано при работе с различными учебниками по актуальным темам.

Примеры.
Какое из двух тел движется с большей скоростью: прохождение 20 м за 10 с или за 4 с — 16 м? Какой из двух кузовов преодолеет большее расстояние за 0,5 ч: движется со скоростью 36 км / ч или 12 м / с?

Трактор проехал путь 500 м за время равное 4 минутам, а в следующие 10 минут — 2 км.Определите среднюю скорость трактора за все время поездки.

Мотоциклист преодолел расстояние между двумя населенными пунктами за 30 минут, двигаясь со скоростью 10 м / с. Сколько времени ему потребуется, чтобы вернуться, если он будет двигаться со скоростью 15 м / с?

Первый 50-метровый трамвай двигался со скоростью 5 м / с, а следующие 500 м — со скоростью 10 м / с. Определите среднюю скорость трамвая в пути.

Как долго пассажир, сидящий у окна поезда, движущегося со скоростью 54 км / ч, будет видеть проезжающий мимо встречный поезд, скорость которого составляет 72 км / ч, если его длина составляет 150 м?

Содержание
Предисловие 3
Учебные задания
Введение 5
TK-1.Измерение физических величин. Точность и погрешность измерения 5
Исходные сведения о строении вещества 6
ТК-2. Строение вещества 6
Взаимодействие тел 8
ТК-3. Механизм 8
ТК-4. Инерция. Взаимодействие тел. Масса корпуса 13
ТК-5. Плотность вещества 15
ТК-6. Явление гравитации. Гравитация 17
ТК-7. Прочность эластичности. Масса тела 19
ТК-8. Сила трения 20
ТК-9. Графическое изображение сил.Добавить сил 21
ТК-10. Силы в природе. Задачи расчета 21
Давление твердых тел, жидкостей и газов 23
ТК-11. Давление твердых тел 23
ТК-12. Давление жидкостей и газов. Закон Паскаля 24
TK-13. Давление жидкости 25
ТК-14. Сообщающиеся суда 26
ТК-15. Атмосферное давление 27
ТК-16. Давление в жидкостях и газах. Расчетные задачи 28
ТК-17. Архимедова сила. Плавательные тела 30
Работа и сила. Энергия. Механизмы простые 32
ТК-18.Механические работы 32
ТЗ-19. Мощность 32
ТЗ-20. Энергетика 33
ТЗ-21. Простые механизмы. КПД механизмов 34
Самоконтроль
ТС-1. Строение вещества 38
ТС-2. Механизм механический 40
ТС-3. Инерция. Взаимодействие тел. Масса тела. Плотность вещества 44
ТС-4. Силы в природе 47
ТС-5. Твердое давление 50
ТС-6. Давление в жидкостях и газах 53
ТС-7. Архимедова сила. Плавательные тела 56
ТС-8. Механическая работа и мощность 60
ТС-9.Энергетика 63
ТС-10. Простые механизмы. КПД простых механизмов 64
Самостоятельная работа
СР-1. Строение вещества 68
СР-2. Механизм 69
СР-3. Инерция. Взаимодействие тел. Масса тела 72
СР-4. Плотность вещества 74
СР-5. Силы в природе 76
СР-6. Давление твердых тел 79
CP-7. Давление в жидкостях и газах 82
СР-8. Архимедова сила. Плавательные тела 84
CP-9. Механическая работа. Мощность 86
СР-10.Энергетика 88
CP-11. Простые механизмы. КПД простых механизмов 89
Контрольные работы
КР-1. Механическое движение. Плотность вещества 92
КР-2. Давление твердых тел, жидкостей и газов 96
КР-3. Архимедова сила 100
КР-4. Механическая работа и мощность. Простые механизмы 104
Примеры решения типовых задач
Механическое движение 108
Взаимодействие тел. Масса тела. Плотность вещества 109
Масса тела. Давление твердых тел 110
Давление жидкостей и газов 111
Атмосферное давление.Архимедова сила 112
Механическая работа и сила 114
Простые механизмы. Работоспособность механизмов 115
Ответы
Практические задания 117
Тесты самоконтроля 117
Самостоятельная работа 119
Контрольные работы 120
Литература 122.


Скачать бесплатно электронную книгу в удобном формате, смотреть и читать:
Скачать книгу Физика, 7 класс, Учебное пособие, А. Э. Марон, Е. А. Марон, 2013 — fileskachat.com, быстро и бесплатно.

Скачать файл No.1 — pdf
Скачать файл №2 — djvu
Ниже вы можете купить эту книгу по лучшей сниженной цене с доставкой по всей России.

Составили: учитель физики
Средняя общеобразовательная школа №3 Еремеева О.А.

Диктанты, способствующие развитию ассоциативного мышления и лучшему усвоению учебного материала.

Тип физической диктовки I

Выберите из перечисленных понятий единицы измерения, физические величины, устройства, явления.Представьте ответ в виде таблицы:

метр, длина, путь, линейка, м / с, килограмм, весы, инерция, спидометр, скорость, время, взаимодействие;

сила, динамометр, диффузия, Ньютон, динамометр, дина, масса, сила тяжести, килограмм, кН, вес, сила тяжести;

плотность, стакан, объем, кг / м 3, вес, весы, килограмм, линейка, г / см 3, инерция;

давление, Паскаль, площадь, тонна, динамометр, Н, диффузия, м 2, Ньютон, сила, килопаскаль, шар Паскаля;

барометр, высота, манометр, гектопаскаль, плотность, сообщающиеся сосуды, высотомер, магдебургские полушария, г / см 3;

плавание, архимедова сила, объем, ньютон, кг / м 3, плотность, масса, вес, ареометр, гидравлический пресс, тормоза, стакан;

работа, джоуль, динамометр, сила, путь, Н, кДж, время, секунда, мощность, киловатт, сила тяжести;

рычаг, момент силы, подвижный блок, плечо силы, метр, шкала, миллиграмм, линейка, см, винт, инерция, А;

работа, сила, кинетическая энергия, масса, кг, E p, кДж, маятник, F, рычаг, барометр, анероид;

Физические диктанты типа II

1) Выберите из перечисленных понятий, слов, фраз, связанных с явлением диффузии и инерции.Ответ представлен в виде таблицы:

2) Выберите из перечисленных понятий, слов, фраз, связанных с явлением силы тяжести и смачивания.

Исаак Ньютон, «без утиной воды», масса тела, 9,8 Н / кг, приливы и отливы, Галилео Галилей, Пизанская башня, роса, мыло, «мокрый как цыпленок», яблоко.

3) Выберите из перечисленных понятий, слов, словосочетаний, связанных с явлением: давление, плавание.

Шар Паскаля, «торричелевская пустота», «камень до дна», подводная лодка, «Магдебургские полушария», ватерлиния, подъемная сила, «Эврика!», Архимед, артезианская скважина.

4) Выберите из перечисленных понятий, слов, фраз, относящихся к: работе, власти.

Путь, «лошадиные силы», мощность, отрицательный, скорость выполнения, положительный, время, ватт, джоуль, двигатель,

5). Выберите из перечисленных понятий, слова, фразы, связанные с: энергией, простыми механизмами.

Лук с натянутой тетивой, маятник, плотина гидроэлектростанции, шар, закон сохранения, Архимед, точка опоры, силовое плечо, маятник, блок, твердое тело, потенциал, кинетический

Физический диктант III типа

Завершить предложение или вставьте пропущенные слова.

Тема: Структура материи

    Молекулы состоят из … (атомов)

    Молекулы во всех телах движутся … (непрерывно и нерегулярно)

    Молекулярная структура вещество подтверждается явлением … (диффузия)

    Молекулы одного и того же вещества … друг от друга. (они не различаются)

    Когда вещество нагревается, объем молекул… (не меняется)

    Молекулы холодной и горячей воды … отдельно. (не различаются)

    Проникновение молекул одного вещества в пространства между молекулами другого вещества называется … (диффузией)

    При той же температуре скорость диффузии минимальна в. .. (в твердом теле)

    Молекулы твердого тела не разлетаются из-за действия межмолекулярных сил … (притяжение)

    Движение частиц краски в воде — пример… (Броуновское движение)

Тема: Взаимодействие тел.

Инерция, плотность

    Явление сохранения скорости телом при отсутствии воздействия на него другими телами называется … (инерцией)

    Если другие тела не действуют на тело или действия других тел уравновешены, тогда тело движется … (равномерно)

    Когда человек спотыкается, он падает … потому что его ноги… и его тело … (вперед, стоп, продолжает движение по инерции

    С … лодку легче спрыгнуть с берега. (с грузом)

    Величина, равная соотношению вес тела к его объему называется … (плотностью)

    Чем ближе молекулы в веществе, тем больше его плотность … (подробнее)

    Если сено спрессовано в тюк, то масса сена, содержащегося в сене … (не изменится)

    Если в стакан налить две несмешивающиеся жидкости, то они будут наверху… густая жидкость. (Меньше)

    Из двух тел одинаковой массы объем тела с большей плотностью … (меньше)

    Из двух тел одинакового объема масса этого тела меньше, плотность … (подробнее)

Сила, давление

    Сила, с которой все тела притягиваются к земле, называется … (гравитация)

    изменяет направление движения камня, брошенного горизонтально.(гравитация)

    Гравитация прямо пропорциональна … (массе тела)

    Сила, с которой тело действует на горизонтальную опору или вертикальную подвеску, называется силой притяжения тела к земле. .. (вес)

    Приливы и отливы являются доказательством явления … (сила тяжести)

    Физическая величина, равная отношению силы, действующей перпендикулярно поверхности, к площади \ u200b \ u200b эта поверхность называется… (давление)

    Если увеличивать площадь поверхности, с постоянной силой, то давление … (уменьшается)

    Для уменьшения давления на почву в автомобилях, тракторах, комбайнах марки .. . (широкие колеса и гусеницы)

    Для уменьшения силы резания необходимо … давление, для этого нужно … площадь лезвия, т.е. … (усилить давление, уменьшить, затачить)

    Чтобы помочь человеку провалиться под лед, нужно подползти к нему, чтобы… давление на лед. (Уменьшить)

Тема: Давление жидкостей и газов.

    Давление, приложенное к жидкости или газу, передается … (без изменения каждой точки жидкости или газа)

    Размеры мыльного пузыря под давлением вдуваемого в него воздуха увеличиваются одинаково во всех направлениях, в результате чего пузырек принимает форму шара. Это явление подтверждает закон … (Паскаль)

    Давление жидкости на одном уровне во всех точках… (то же)

    В сообщающихся сосудах различной формы однородная жидкость устанавливается на … (один уровень)

    В невесомости зубная паста выдавливается из тюбика … (будет be)

    В морской воде … плавать, чем в речной воде. (Проще)

    К весам подвешены два груза одинаковой массы — один алюминиевый, другой железный, если одновременно поместить гири в сосуд с водой, то весы весы… (сломано)

    Сила плавучести всегда направлена ​​… (вверх)

    Когда судно движется из реки в море, его осадка … (уменьшается)

Тема: Работа и власть.

    Величина, равная произведению силы на …, называется работой. (Путь)

    Единица измерения механической работы в СИ называется … (джоуль)

    Работа может быть … и … (положительной, отрицательной)

    Когда тело движется по горизонтали работа — это сила тяжести… (равно нулю)

    Гиря неподвижно висит на тросе, при этом механическая работа … (не выполняется)

    Бочка наполнена водой. С помощью ведра девушка зачерпнула половину воды из бочки, а мальчик, который отстал. Мальчик действительно … работал. (большой)

    Мощность — это величина, которая показывает … (как быстро выполняется работа)

    Ватт — единица мощности в системе СИ равна отношению … (джоуль в секунду)

    Чем больше работы выполняется в единицу времени, тем… мощность. (Подробнее)

    механическую работу выполняют мальчики одинакового веса, поднимаясь по лестнице на одинаковую высоту, один за 1 минуту, другой за 40 секунд? (неравные)

Тема: Простые механизмы и энергия.

    Жесткое тело, которое свободно вращается вокруг фиксированной точки опоры, называется … (рычаг)

    Подвижный рычаг дает прирост прочности … раз. (два)

    рычаг не набирает силу.(неподвижно)

    Рычаг находится в равновесии, если момент силы … (вращение его по часовой стрелке равно моменту сил, вращающихся против часовой стрелки)

    Для уменьшения прилагаемой силы требуется … плечо сила. (Увеличить)

    Ни один из простых механизмов не дает выигрыша в … (работе)

    Потенциальная энергия тела, поднятого над землей, может быть увеличена, если: … (увеличить массу тела или поднять кузов на большую высоту)

    Если уменьшить скорость тела, то его… энергия уменьшится. (кинетическая)

    Потенциальная энергия речной воды … в ее источнике, а не в устье. (Подробнее)

    Энергия никуда не исчезает и не возникает ни из чего, она только … от одного вида к другому. (поворачивает)

Тема: Исходная информация о структуре веществ

Есть три типа заданий, которые можно выполнить с кроссвордами:

Тип I.

Расставьте числа для решенного кроссворда и ненумерованные вопросы для кроссворда.

    сокращенная запись одной доли метра — 0,01 метра. (1)

    наука, изучающая общие законы природных явлений, свойства и структуру материи, законы ее движения (3)

    мельчайшая частица данного вещества. (4)

    единица измерения температуры. (девять)

    объясняется тем, что молекулы жидкости сильнее притягиваются друг к другу, чем к молекулам твердого тела.(6)

    самое удивительное вещество на Земле. (2)

    физическое явление адгезии молекул жидкости и твердого тела. (пять)

    одним из методов исследования является источник физических знаний. (7)

    частицы, из которых состоят молекулы. (десять)

    сокращенная запись 0,001 метра. (одиннадцать)

    взаимное проникновение веществ, контактирующих друг с другом, происходящее из-за беспорядочного движения частиц вещества. (восемь)

II типа.

Для этих вопросов заполните кроссворд

    Сокращенная запись одной доли метра — 0,01 метра.

    Самое удивительное вещество на Земле.

    Наука, изучающая общие законы природных явлений, свойства и структуру материи, а также законы ее движения.

    Наименьшая частица данного вещества.

    Физическое явление адгезии молекул жидкости и твердого тела.

    Это объясняется тем, что молекулы жидкости сильнее притягиваются друг к другу, чем к молекулам твердых тел.

    Одним из методов исследования является источник физических знаний.

    Взаимное проникновение контактирующих веществ друг в друга, происходящее за счет беспорядочного движения частиц вещества.

    Единица измерения температуры.

    Частицы, из которых состоят молекулы.

    Сокращенный ввод 0,001 метра.

III тип.

Используя заполненный кроссворд, определите каждый термин

    см — (сокращенная запись одной доли метра — 0,01 метра.)

    вода — (самое удивительное вещество на Земле)

    физика — (наука который изучает общие законы природных явлений, свойства и структуру материи, а также законы ее движения)

    молекула — (мельчайшая частица данного вещества)

    смачивание — (физическое явление адгезии молекул жидкость и твердое тело)

    несмачивание — (объясняется тем, что молекулы жидкости больше притягиваются друг к другу, чем к твердым молекулам)

    наблюдение — (один из методов исследования является источником физических знаний)

    диффузия — (взаимное проникновение веществ, контактирующих друг с другом, которое происходит из-за случайного движения частиц вещества)

    градуса — (единица температуры)

    атом — (частицы т. t составляют молекулы)

    мм — (сокращенная запись 0.001 метр)

2. Взаимодействие тел

    Действие одного тела на другое.

    Один из способов познания

    Определение массы тела с помощью силы тяжести.

    Воображаемая линия, по которой движется тело.

    Физическая величина, равная отношению массы тела к его объему.

    Физическая величина, равная отношению пройденного расстояния ко времени поездки.

    Явление сохранения скорости телом при отсутствии на него воздействия со стороны других тел.

    Килограммы, метры и другие единицы измерения хранятся в городе Севр недалеко от Парижа.

    Значение, измеряемое в секундах, минутах и ​​других единицах измерения.

    Длина траектории.

3. Давление твердых тел, жидкостей и газов

    [грузоподъемность] — вес груза, перевозимого судном.

    [воздухоплавание] — состояние равновесия тела, погруженного в газ.

    [лифт] — разница между весом воздуха и весом того же объема газа.

    [водоизмещение] — сила тяжести, действующая на судно с грузом.

    [Блез Паскаль] — французский ученый, математик, физик, философ.

    [Отто Герике] — ученый, проводивший эксперимент с «Магдебургскими полушариями».

    [ватерлиния] — линия на корпусе, показывающая.

    [атмосфера] — воздушная оболочка земли.

    [барометр] — прибор для измерения атмосферного давления.

    [Архимед] — древнегреческий ученый, физик и математик.

    [осадка] — глубина погружения судна в воду.

    [объем] — физическая величина.

    [метр] — основная единица измерения длины в системе СИ.

    [вес] — сила, действующая на горизонтальную опору или вертикальный подвес вследствие притяжения тела к Земле.

    [Па] — основная единица измерения давления в системе СИ.

    [Н] — основная единица силы в СИ.

Работа и власть. Энергия.

    [потенциальная энергия] — энергия, которая определяется взаимным расположением взаимодействующих тел или частиц одного и того же тела.

    [подвижный блок] — блок, дающий прирост силы в 2 раза.

    [блок] — это своего рода рычаг.

    [мощность] — величина, характеризующая скорость работы.

    [механизм] — устройство, используемое для преобразования силы.

    [силовое плечо] — кратчайшее расстояние между точкой опоры и прямой линией, по которой сила действует на рычаг.

    [Ватт] — единица измерения мощности в СИ.

    [Джеймс Джоуль] — английский физик, один из первооткрывателей закона сохранения энергии.

    [момент силы] — это произведение модуля силы, вращающей тело плечом.

    [рычаг] — твердое тело, которое может вращаться вокруг неподвижной опоры.

    [КПД] — отношение полезной работы к затраченной.

    [статика] — раздел механики, изучающий состояние равновесия тел под действием сил.

    [закон] — внутренняя и необходимая сущностная связь между предметами и явлениями объективной реальности.

    [кДж] — 1000 Дж.

    [с] — единица времени в системе СИ.

Power

    [весы] — прибор для измерения веса тела с использованием силы тяжести.

    [трение] — взаимодействие между твердыми телами, возникающее при движении и контактирующих телах, по поверхности и контакту.

    [динамика] — раздел физики, изучающий движение тел под действием сил.

    [динамометр] — прибор для измерения силы.

    [Исаак Ньютон] — английский ученый, создатель классической физики.

    [сила упругости] — сила, возникающая в твердом теле при деформации.

    [гравитация] — сила, с которой Земля притягивает тело к себе.

    [деформация] — изменение формы или размеров твердого тела.

    [движение] — способ существования материи.

    [Ньютон] — единица силы в СИ

    [сила] — векторная величина — мера взаимодействия тел.

ГДЗ по физике Прыскина Упражнение 17.

  • Понимание основ мироздания — основные необходимые знания для ребенка, которые станут основой успеха практически во всех сферах жизни. Именно систематическое овладение всеми аспектами науки, доскональное изучение даже мельчайших нюансов материала — основная цель, которой поможет выполнить готовое домашнее задание по физике 7 класса Пони, в полном объеме, предложенном на этом ресурсе.
  • Очевидная экономия времени, которого школьникам постоянно не хватает из-за большой тренировочной нагрузки, обеспечиваемая обращением к материалам Решебника, сопровождается несомненной пользой от пошагового и методически выверенного объяснения всех решений и вопросов. Конечно, полезно, когда можно сослаться на готовый ответ и узнать, был ли ход мысли верным, но еще продуктивнее то, что GDZ Physics 7 Pryony класс содержит точно доступное объяснение Все этапы задач.Такой подход позволяет не только качественно выполнить. Но даже дополнительно повторять весь материал на уроках, причем в том темпе, который удобен каждому ученику. При необходимости вы можете вернуться к родственным или пропущенным темам, чтобы уверенно ориентироваться в сложном материале.
  • Основы физики для семиклассников

  • Начала физики изучают школьники седьмого класса. Знания о знаниях, необходимых для освоения курса по данной дисциплине, закладывались еще в начальной школе, при изучении предмета мира.Программа по физике для секторов достаточно сложная, вмещает большой объем разнопланового материала — новые понятия, законы, термины. Для того, чтобы разбираться в физике и интересоваться ею, нужен ответственный подход к такой работе и качественная, интересная литература, решебники к ней. Подобрать подходящий материал можно самостоятельно либо с помощью родителей, либо обратившись к профессионалам. Среди них — репетиторы по физике, школе и не только, учителя, руководители кружки.
  • Обналичивание с помощью GDZ , Важно учесть:
    — Автогол.Кто-то планирует поднять оценку, текущий и итоговый балл, кто-то — подготовиться к олимпиадам и соревнованиям по физике;
    — Информация о багаже, доступная на старте. Это позволит составить более эффективный и грамотный план, который можно будет корректировать по мере выполнения результатов;
    — необходимость регулярно оценивать результаты, за определенный период или по окончании изучения темы, блокировать темы;
    — Функции, которые предоставляют такие классы.Например, порядок правильной записи результата, который видят семиклассники, работающие на ГДЗ Регулярно, автоматически запоминается и позволяет не допускать досадных ошибок неправильного ответа.
  • Среди интересных и полезных учебных материалов специалисты называют пособие по физике для 7 класса, составленное Прибиным А.В. Базовый учебник отличается ясностью и ясностью материала, что крайне необходимо на первых порах, пока у последовательностей не сформируются представления и понятия, характеризующие физические Объекты и явления.Качественные иллюстрации — еще один плюс, который отмечают и школьники, и учителя.
  • Чтобы сделать подготовку более плодотворной и эффективной, в базовый учебник Приракина можно добавить и другие пособия того же автора. Например, рабочие тетради, сборники практических, контрольных и дидактических работ по физике, тетрадь для лаборатории, контрольные материалы, вспомогательные рефераты, а также многоуровневые задания для секторов по дисциплине.

Начните вводить часть условия (например, может ли она быть или найдена):

Глава I Законы взаимодействия и движения тел.Упражнение 17.

  • 1. Мяч катился по горизонтальной поверхности стола из точки А в точку на (рис. 35). В точке мяча, питаемой силой F. В результате он начал двигаться к точке C. В каком из направлений, отмеченных стрелками 1, 2, 3 и 4, могла действовать сила f?
  • 2. На рисунке 36 показана траектория движения мяча. На ее кружках отмечены позиции мяча через каждую секунду после начала движения. Действовала ли сила на участке 0-3 на мяч; 4-6; 7-9: 10-12; 13-15; 16-19? Если бы сила действовала, как бы она
  • 3 *.На рисунке 37 линия ABCDE показывает траекторию движения некоторого тела. В каких областях тела, вероятно, действовала сила? Может ли какое-либо усилие действовать на тело при его движении на других участках этой траектории? Все ответы оправдывают.

В 9 классе на школьников колоссальная нагрузка, и физика играет в этом последнюю роль. В этот период времени студенты изучают такие темы, как законы взаимодействия и движения тел, механические колебания и волны, звук, электромагнитное поле, строение атома и ядра.К каждому из разделов нужно подойти серьезно. Кроме того, некоторые студенты выбирают его в качестве предмета для сдачи экзамена ОГЭ.

В подавляющем большинстве школ применяется классический учебник на предмет авторства Перешкина А.В. и Джокер Э.М.Эти методисты известны своими обучающими преимуществами, которым учились миллионы людей. Помимо подробного теоретического материала в книге также есть вопросы после абзацев и упражнения для закрепления знаний.Часто ученики испытывают трудности с поиском ответов и решением задач. В таких случаях могут помочь решебников по физике для 9 класса (авторы: Пририкин А.В. и Хотник Е.М.) с готовыми ключами.

Как устроен сборник ГДЗ и как правильно им пользоваться?

Пособие содержит в себе как подробные алгоритмы поиска ответов на задачи, так и пояснения по вопросам, стоящие после абзаца. Чтобы найти необходимую информацию, достаточно найти свой номер.Помимо прочего, есть также вспомогательные материалы для лабораторных занятий и раздел для самотестирования.

Перед просмотром данной информации девятикласснику рекомендуется самостоятельно попытаться решить задачу. После этого вы можете связаться с ключами и сравнить результаты. Все примеры соответствуют GEF, поэтому в их правильности сомневаться не приходится.

Чем может помочь готовое домашнее задание?

Это издание предназначено для школьников, которые, возможно, не слишком хорошо разбираются в предмете, но хотят иметь хорошие отметки.В этом им поможет инструкция:

  • качественно проанализировать свою деятельность и уровень знаний;
  • заполнить пропуски в пропущенном материале;
  • улучшают средний балл по дисциплине.

вариантов физических тестов для задач.среднее профессиональное образование

Любое явление материального мира может быть изучено экспериментальной и теоретической физикой. Физические открытия стимулировали прогресс человечества в целом. Благодаря исследованиям в свое время появились телефоны, машины, компьютеры и другие полезные и важные изобретения.

Физика как наука о мире и его законах всегда была двигателем прогресса. Конечно, гуманисту не нужно глубокое погружение в предмет, но взрослый без базовых знаний производит странное впечатление, когда не понимает критериев выбора автомобиля и других элементарных вещей.

Знание физики пригодится хотя бы для того, чтобы не чувствовать себя Алисой в стране чудес. Невежество наполняет мир тайнами и загадками, но взрослый человек, не понимающий, почему ночью видны звезды, утром появляется роса, а колеса резиновые, вызывает симпатию.

Неподвижные заряженные частицы создают электростатическое поле. Движущиеся частицы образуют магнитное поле, то есть электрический ток. Электрические и магнитные поля тесно связаны, поскольку они порождают друг друга.Без этой связи не было бы ни радиоволн, ни света.

Физика как наука постоянно развивается, все глубже и глубже погружаясь в теорию относительности, квантовую механику и электродинамику, изучение атомной энергии, гидродинамику, аэродинамику. Новые знания позволяют человеку постичь секреты собственного бытия и бесконечного пространства.

Электростатика изучает взаимодействие стационарных электрических зарядов. Одинаково заряженные тела отталкиваются, а противоположные, наоборот, притягиваются — это физическое явление знакомо каждому школьнику.Школьный курс по электростатике углубляет понимание этого процесса.

Водяной пар попадает в атмосферу в результате испарения, скорость которого зависит от температуры воздуха и ветра, рельефа, растительного покрова и т. Д. На земле постоянно происходят два противоположных процесса: поверхность орошается за счет атмосферных осадков и осушается за счет испарения. .

Системы автоматизированного проектирования реализуют технологии проектирования. Система САПР состоит из персонала и ряда технических, программных и других средств автоматизации.Система была разработана в США после Второй мировой войны для военно-промышленного комплекса.

Электротехника занимается производством, распределением, преобразованием и использованием электроэнергии, а также проектированием и эксплуатацией компонентов, цепей и оборудования. Как наука об электротехнике изучает электрические и магнитные явления для практического использования.

Ученые с помощью различных приборов пытаются описать принципы процессов, происходящих в природе планеты и Вселенной.Научные теории склонны синтезировать факты и наблюдения за конкретными явлениями. Не все теории сводятся к уравнениям, но в физике они успешны.

Гений Альберта Эйнштейна — доказанный факт, и все, что мы узнаем о выдающейся физике, только подтверждает его необычность. История жизни ученого наполнена неожиданными и сложными событиями, знакомство с которыми удивляет.

Физика — это естествознание, изучающее общие характеристики всего, что нас окружает.Предмет включает изучение материи (материи и поля) и различных форм ее существования и движения. Область изучения физики — это также основные природные явления, которые управляют движением материи. Основная задача физики — выявить законы природы, лежащие в основе всех физических явлений, с целью использования их на практике.

Изучение мира ясно показывает, что он познаваемый, но бесконечный в знании. Материя находится в постоянном движении, то есть постоянно изменяется.Таким образом, мир — это материя в постоянном движении и развитии. Физические законы — это общие законы для всех материальных систем.

Физика считается фундаментальной наукой, поскольку для других естественных наук (биологии, химии, геологии и др.) Физические законы являются базовыми для описания изучаемых материальных систем. Например, химия изучает превращение одних веществ в другие. Химические свойства веществ в этом случае определяются свойствами атомов и молекул, описываемых физическими законами — термодинамикой, электромагнетизмом, квантовой физикой.

Тест по физике № 1

Тест по главе «Введение» и пунктам 1-6 учебника Физика. 7 классы: учеба на общеобразовательную. учреждений / А.В. Перышкин. — 11-е изд., Стреотип. — М .: Дрофа, 2007. — 192 с .: ил.

Диагностическая работа позволяет оценить достижение важнейших запланированных результатов в соответствии с содержанием курса физики 7 класса по разделу «Введение». «.

Инструкции по тестированию

Тест занимает 15 минут.Вариант диагностической работы состоит из 11 вопросов с множественным выбором. При прохождении теста вы должны выбрать один правильный ответ на каждый вопрос. Задания 1–6, 11 оцениваются в 1 балл, 7–10 — в 2 балла и 0 баллов, если отсутствует элемент правильного ответа. Максимальный балл за всю работу — 15 баллов. Шкала перевода баллов в оценку: 13-15 баллов (5), 10-12 баллов (4), 7-9 баллов (3), 6 и менее баллов (2).

Демо-вариант

1. Называются ли они физическими явлениями?

a) любые преобразования вещества,

b) любое проявление свойств вещества,

c) любое преобразование вещества или проявление его свойств без изменения состава вещества,

d) любое трансформация вещества или проявление его свойств при изменении состава вещества.

2. Укажите физическое тело.

а) стекло, б) алюминий, в) дерево, г) окно.

3. Какое из представленных выражений обозначает физическую величину:

а) глубина реки, б) вода в реке, в) чистая вода, г) холодная вода.

4. В задаче перечислены физические величины и их единицы. Пожалуйста, выберите истинное соответствие .

а) время, метр (м), б) длина, секунда (с),

в) температура, миллиметр (мм), г) объем, кубический метр (м3).

5. В каком случае ниже фигурирует человек, проводящий эксперимент?

а) когда он сидит на берегу озера и смотрит, как лодка удаляется от него,

б) когда он стоит на берегу и вынимает из воды упавшую в него монету,

в) когда он бросает в реку камешки, фишки, кусочки бумаги и смотрит, какой из этих предметов утонет,

d) когда он наблюдает за облаками.

6. Ниже приведены числовые обозначения префиксов.Какой ответ ошибся ?

а) сенти (в) — 0,01, б) милли (м) — 0,001,

в) микро (мк) — 10000, г) кило (к) — 1000.

7. Экспресс 10 мл в см3, дм3, м3.

а) 100 см3; 0,01 дм3; 0,1 м3, б) 100 см3; 1000 дм3; 10000 м3,

в) 10 см3; 0,01 дм3; 0,00001 м3, г) 10 см3; 1 дм3; 0,01 м3.

8. Какими будут значения длин, равных 4 м и 100 м, если они выражены в см?

а) 40 см и 10 см, б) 400 см и 1 см,

в) 400 см и 10 см, г) 40 см и 1 см.

9. Определите показания секундомера.

а) 42 мин 7 с, б) 49 с, в) 7 ч 42 мин, г) 7 мин 42 с.

10. На рисунке изображен карандаш, плотно обвитый тонкой проволокой, и линейка. Определите деление шкалы и диаметр проволоки.

а) 1 мм; 2 мм. б) 1 см; 2 мм, в) 1,2 см; 3,6 см, г) 1 см; 2,4 см.

11. Почему всем нужно знать физику?

а) потому что физика объясняет причины различных природных явлений,

б) эта наука позволяет вам создавать новую, все более совершенную технику,

в) потому что физика дает знания о самых общих законах природы, которые играют большую роль в жизни каждого человека,

г) все ответы верны.

Правильные ответы

Министерство общего и профессионального образования

Промежуточный аттестат

Разработчик

Учитель 1 категории

Лоренц С.Ю.

Н-Тагил

2012

Пояснительная записка.

    Положения:

    Закон РФ «Об образовании»

    Федеральный закон «О внесении изменений и дополнений в статью 16 Закона Российской Федерации« Об образовании »от 20.07.2000 № 102-Ф3.

    Концептуальный подход к формированию «Основных направлений развития образования в Свердловской области на 2004-20007 годы (Программа IV)».

    Государственный образовательный стандарт для общественных организаций.

    Программы развития образовательных учреждений.

    Учебная программа OU.

    Положение о промежуточной и итоговой аттестации студентов ПУ №104.

    Пример программы в теме.

Цель: Определить соответствие навыков и умений обучающихся государственному стандарту

Задания:

1.Оцените навыки и умения, полученные при изучении предмета

2. Проанализируйте факторы, влияющие на качество обучения.

3. Отслеживать динамику развития качества образования.

В результате освоения учебной дисциплины обучающийся должен уметь:

    описывать и объяснять физические явления и свойства тел: движение небесных тел и искусственных спутников Земли; свойства газов, жидкостей и твердых тел;

    различаются гипотезы из научных теорий;

    сделать выводы на основе экспериментальных данных;

    приведите примеры, чтобы показать, что: наблюдения и эксперимент являются основой для выдвижения гипотез и теорий, позволяют проверить истинность теоретических выводов; физическая теория позволяет объяснять известные природные явления и научные факты, предсказывать еще неизвестные явления;

    приведу примеры практического использования физических знаний: законы механики, термодинамики

    воспринимать и на основании полученных знаний самостоятельно оценивать информация, содержащаяся в сообщениях СМИ, Интернете, научно-популярных статьях.

    применять полученные знания для решения физических проблем;

    определить характер физического процесса по расписанию, таблице, формуле;

    для измерения ряд физические величины, представляющие результаты измерений с учетом их погрешностей;

В результате освоения учебной дисциплины студент должен знать:

    значение понятий: физическое явление, гипотеза, закон, теория, вещество, взаимодействие

    значение физических величин: скорость, ускорение, масса, сила, импульс, работа, механическая энергия, внутренняя энергия, абсолютная температура, средняя кинетическая энергия частиц материи, количество тепла

    смысл физических законов классическая механика, всемирное тяготение, сохранение энергии, импульс

    вклад российских и зарубежных ученых это оказало наибольшее влияние на развитие физики;

Форма аттестации: Тест

Методология: письменный

Инструменты: Контрольные задания, творческое задание

Метод оценки: эксперт

Раздел: Механика, молекулярная физика

000

Понимать сущность и социальную значимость своей будущей профессии, проявлять к ней устойчивый интерес

Организовывать свою деятельность исходя из цели и методов ее достижения, определенных руководителем

Анализировать рабочую ситуацию, выполнять текущую и окончательную контролировать, оценивать и корректировать свою деятельность, нести ответственность за результаты своей работы.

Поиск информации, необходимой для эффективного выполнения профессиональных задач

Использование информационных и коммуникационных технологий в профессиональной деятельности

Работа в команде, эффективное общение с коллегами, руководством, клиентами

Прохождение военной службы, в том числе с использованием полученных профессиональных знаний ( для мальчиков)

Критерии компетенций (базовые, общие)

компетенции

Общие компетенции

Тематические критерии

Номера должностей

ОК 7

Необходимость выполнять работу;

Показывает эмоциональную и психологическую устойчивость;

Регулирующие компетенции

Знать основные понятия

Знать основные формулы

Знать основные законы

Выполнять задания по образцу

Социальные компетенции

Использует справочную литературу;

Умеет пользоваться таблицами и графиками

Аналитические компетенции

Выполнять задачи, отличные от алгоритма

Творческие компетенции

Найдите задачу в тексте

Сделайте небольшую заметку

Решение задачи

Самостоятельно повышение компетенций

Проверить формулу на размерность

Самооценка

Промежуточные вычеты

Самоуважение

Промежуточная контрольная работа 1 курс

    Какие силы в механизме сохраняют свое значение при переходе от одного обратного отсчета инерционной системы к другому ?

    Только гравитационный.

    Только силы упругости.

    Только силы трения.

    Все три силы, указанные в ответах AB.

    Значения всех трех сил изменяются

    Почему в системах отсчета, связанных с Землей, в большинстве практических случаев невидимы явления, связанные с неинерционностью систем отсчета?

    Потому что Земля вращается равномерно с постоянным периодом обращения.

    Потому что центростремительные ускорения точек на поверхности Земли малы.

    Потому что период вращения Земли вокруг своей оси большой.

    Потому что центростремительное ускорение точек на поверхности Земли намного меньше ускорения свободного падения.

    Среди ответов A – D нет правильного ответа.

    Диск radiusR катится по горизонтальной поверхности без проскальзывания. В данный момент скорость центра диска (точка O) относительно Земли равна V (рис. 1). Каковы скорость и ускорение точки A в системе отсчета, связанной с Землей?

И. В, В / Р

Б. 2В, В / Р

IN. 2 В, 4 В / R

Д. 2В, В / 2Р

Д. В, В / 2Р

4. В трех сосудах объемом V = V = V = V при той же температуре существуют химически нереагирующие газы с массами, м, m и давления p, п, p соответственно. Газы закачивались в один сосуд объемом V … Какое из следующих равенств не верно?

1).m = m + m + m 2) .p = p + p + p 3) .V = V + V + V

5. При каком условии A = 0?

1). Т = 0 2). р = 0 3). V = 0

6. Температура кипения жидкости:

1). При повышении атмосферного давления

2). При повышении атмосферного давления

3). Не зависит от атмосферного давления

7. Среднее расстояние между молекулами воды при переходе воды из газообразного состояния в твердое состояние при нормальном давлении уменьшится примерно на…

А. 10 раз. Б. 20 раз.

Б 100 раз. Г. 1000 раз.

D. 10000 раз.

8. Как объем этого количества идеального газа притягивается из состояния 1 для состояния 2?

A. Увеличено.

Б. Без изменений.

B. Уменьшено.

г. Может увеличиваться или уменьшаться для шитья.

E. Процесс невозможен.

9. На рисунке показана зависимость давления одного моля идеального газа от объема.Какой из графиков может соответствовать в зависимости от давления газа и температуры

Для такого процесса?

D. Ни одного графика.

10. В некоторых процессах, выполняемых с идеальным газом, соотношение между давлением и объемом газа pV 2 знак равно const … Как изменится температура T на га, если увеличить ее объем в 2 раза?

А. Менять не буду.

Б. Увеличится в 4 раза.

B. Уменьшится в 4 раза.
D. Увеличится в 2 раза.
D. Уменьшится в 2 раза.

11. на р- V-диаграмма (рис. 1) показывает пять процессов изменения состояния газа. Какой из графов может соответствовать адиабатическому процессу?

Рис. 1

12. Для измерения влажности атмосферного воздуха в сосуд, содержащий воздух, закапали несколько капель воды, быстро закрыли сосуд пробкой и соединили с манометром воды.В нескольких мичикпихе манометр зафиксировал повышение давления в суде на 10 см водяного столба. Какая относительная влажность окружающего воздуха? Температура воздуха 20 ° C. Давление насыщенной воды при 20 ° C составляет 2,33 кПа.

А. 100%. Б. 42%. H. 58%. Г. 14%. Д. 70%.

Творческое задание: прочитать отрывок сформулировать задачу, сделать небольшую пометку, нарисовать и решить задачу

Ну вот так и идет наш Иван
За кольцом на окиян.
Горбун летит как ветер.
И в инициации в первый вечер
Помахали сто тысяч миль
И нигде не уперся.

Эмоционально-психологическая компетентность

Регуляторные компетенции

Социальные компетенции

Аналитические компетенции

Творческие компетенции

Компетенции самосовершенствования

Всего баллов

Оценка «5» — от 15-18 баллов, «4» — от 11-14 баллов, «3» — от 8-10 баллов, «2» — до 8 баллов.

Памятка школьнику

Работа состоит из 13 заданий:

    • 12 тестовых заданий: в каждом задании выберите один ответ;

      одно творческое задание: прочитать отрывок, сформулировать задачу, сделать небольшую заметку, рисунок и решить задачу

:

Система оценки:

0- критерий не проявляется

1- критерий проявляет себя

Руководство по оценке результатов тестирования

Работа оценивается по следующим параметрам :

    Правильность задания

    Использование справочной литературы

    Самооценка

    проявление эмоциональной устойчивости при выполнении работы

Уровень креативности и самооценки оценивается по таблице критериев и при наличии промежуточных расчетов

Работа занимает 40 минут

Система оценки:

0- критерий не проявляется

1- критерий проявляет себя

Максимальное количество баллов: 18

Базовый уровень — от 8-10 баллов — оценка «3»

Оптимальный уровень — от 10-14 баллов — оценка «4»

Продвинутый уровень — от 15-18 баллов — оценка «5»

Полное наименование _________________________________________ группа _______________

Форма ответа

Номер вопроса

Возможный ответ

Промежуточные расчеты, записи

дано: Рисунок

, чтобы найти

Решение вопроса:

0002 самооценка

000 число

000 ответ

Аналитическая записка по результатам контрольной (практической) работы в ______ группе за _________________________________________ за 20.. — .. академический год.

цель : установление уровня выполнения ФГОС среди обучающихся.

Контрольная (практическая) работа состояла из следующих задач:

_______________________________________________________________________________________________________________________________________________________________________________________________________________________

По результатам контрольной (практической) работы получены следующие результаты:

1. Реализация ФГОС в части формирования общих компетенций:

Студенты проявили владение ________________________________________________

_____________________________________________________________________________
___________________________________________________________________________________________________________________________________________________________________________________________________________________________________________________________________________________________________

2. Реализация ФГОС в части формирования профессиональных компетенций:

Студенты проявили владение _________________________________________________

______________________________________________________________________________________________________________________________________________________________________________________________________________________________________________________________________________________________________________

3.Студенты показали следующий уровень сформированности компетенций (%):

Эмоционально-психологический ______________________________

Нормативный ______________________________

Социальный ______________________________

Образовательно-познавательный ______________________________

Творческий ______________________________

______________________________

результаты контрольной (практической) работы :

Студенты показали владение ФГОС на уровне __________________________________________ на уровне _______________________.

(неверно, приемлемо, оптимально)

Удалось достичь высокого уровня формирования _____________________________________________________________________________.

В дальнейшей работе следует обратить внимание на формирование _____________________________________________________________________________.

Дата: Преподаватель: __________________________

Библиографический список

Нормативно-правовые документы

    Сборник нормативных документов «Общее среднее образование России», Москва, «Прогресс», 1998.

    Закон Российской Федерации «Об образовании» от 10.07.1992 № 3266-1.

    Федеральный закон «О внесении изменений и дополнений в статью 16 Закона Российской Федерации« Об образовании »от 20.07.2000 № 102-Ф3.

    Концептуальный подход к формированию «Основных направлений развития образования в Свердловской области на 2004-20007 годы (Программа IV)».

    Государственный образовательный стандарт среднего (полного) общего образования (федеральный и национально-региональный компоненты).

    Программа развития образовательного учреждения.

    Образовательный план учебного заведения.

    Положение о промежуточной и итоговой аттестации обучающихся ПУ №104.

    Примерная программа по предмету.

10. Громов С.В. Шаронова Н.В. Физика, 10-11: Книга для учителя. — М., 2004.

11. Кабардин О.Ф., Орлов В.А. Задачи экспериментальной физики. 9-11 классы: учебное пособие для учащихся общеобразовательных учреждений.- М., 2001.

12. Касьянов В.А. Методические рекомендации по использованию учебников В.А. Касьянова «Физика. 10 кл. «,» Физика. 11 кл. «При изучении физики на базовом и профильном уровне. — М., 2006.

13. Касьянов В.А. Физика. 10, 11 кл. Тематика и планирование уроков. — М., 2002.

14. Лабковский В.Б. 220 задач физики. с решениями: книга для учащихся 10-11 классов образовательных учреждений. — М., 2006.

15. Таблицы констант

.

Краснодарский гуманитарно-технологический колледж

Рассмотрено на собрании Утверждено

Кафедра общих гуманитарных наук Заместитель директора по ОВО

и естествознанию ____________ Г.А. Словцова

Заведующая кафедрой _________ Яценко Т.С. «____» __________ 20__

«___» __________________ 20__

Варианты тестирования

по дисциплине «Физика»

для студентов СПО

Составлено:

Учитель

Яценко Т.С.

Краснодар 2010

Контрольные задания по физике для учащихся среднего профессионального образования.

Раздел 1 «Механика»

  1. Искусственный спутник обращается вокруг Земли по круговой орбите радиуса R с периодом 1 сутки.Каков путь и движение спутника за 1 день?

D. Путь 2πR, ход 0.

E. Путь πR, ход 0.

E. Путь πR, смещение 2R.

  1. С каким ускорением перемещается штанга массой 10 кг под действием силы 5 Н?

A. 50 м / с 2

B. 25 м / с 2

H 2 м / с 2

G. 0,5 м / с 2

  1. Моторная лодка движется по реке со скоростью 5 м. / с, а в стоячей воде со скоростью 3 м / с.Какая скорость течения реки?

A. 1 м / с

B. 1,5 м / с

V. 2 м / с

D. 3,5 м / с

  1. Если многократно сжимать пружину, то она нагревается, потому что:

A. потенциальная энергия пружины превращается в кинетическую

B. кинетическая энергия пружины превращается в потенциальную

B. часть энергии пружины переходит в ее внутреннюю энергию

D. пружина нагревается при трении против воздуха

  1. Пассажир лифта находится в состоянии покоя относительно земли, если:

А.лифт опускается

B. лифт движется равномерно

B. лифт движется вверх с ускорением 9,8 м / с2

D. при любом из вышеперечисленных условий

  1. Какую из формул можно использовать для расчета кинетической энергии движущегося тела:
  1. Если  s — движение тела за сколь угодно малый промежуток времени t, то какое значение определяется соотношением?

A. Путь

B. переместить

B.Скорость только при прямолинейном движении.

D. Мгновенная скорость любого движения

E. Ускорение

  1. Если обозначить  изменение скорости за сколь угодно малый промежуток времени t, то эта величина определяется соотношением?

A. Увеличение скорости.

B. Уменьшение скорости

B. Ускорение только равномерного движения по окружности.

D. Ускорение любого движения

  1. Автомобиль начинает прямое равномерно ускоренное движение из состояния покоя.Какой путь будет пройден за 1 мин при движении с ускорением 2 м / с2 ?

A. 1 м

B. 2 м

H 120 м

H. 1800 м

D. 3600 м

E. 7200 м

  1. Какое расстояние преодолеет самолет до остановки, если он ускорение при торможении составило 6 м / с2 , а скорость в начале торможения 60 м / с?

A. 600 м

B. 300 м

H.360 м

D. 180 м

  1. Искусственный спутник обращается вокруг Земли по круговой орбите радиуса R с периодом 1 сутки.Каков путь и движение спутника за 12 часов?

A. Путь и смещение одинаковы и равны нулю.

B. Путь и смещение одинаковы и равны 2πR.

B. Путь и смещение одинаковы и равны 2R.

D. Путь 2πR, ход 0.

E. Путь πR, ход 0.

E. Путь πR, смещение 2R.

  1. Если обозначить путь, s — это движение тела за время t,  t и  s — путь и движение тела за сколь угодно малый промежуток времени t, то по какой формуле определяется мгновенная скорость тела?

А./ t

B. s / t

B.  s /  t

G.  /  t

  1. Автомобиль трогается с места прямо, равноускоренное движение из состояния покоя. Какой путь пройдете за 0,5 мин при движении с ускорением 0,4 м / с2 ?

A. 0,05 м

B. 0,1 м

H 12 м

G. 180 м

D. 360 м

  1. Каково расстояние, которое самолет преодолевает до остановки, если его ускорение при торможении составляло 4 м / с2 , а скорость в начале торможения 40 м / с?

А.400 м

B. 200 м

H 160 м

G.80 м

  1. Человек идет со скоростью 5 км / ч относительно вагона поезда в направлении его движения, поезд движется со скоростью 5 км / ч. скорость 20 км / ч относительно Земли. С какой скоростью движется человек относительно Земли?

A. 5 км / ч

B. 20 км / ч

H.25 км / ч

G. 15 км / ч

  1. Каково направление вектора ускорения при равномерном движении тела по окружности ?

А.В направлении вектора скорости

B. Против направления вектора скорости

B. К центру окружности

D. Из центра окружности.

E. Ускорение нулевое.

17. На повороте автомобиль движется по окружности радиусом 10 м с постоянным модулем упругости 5 м / с. Что такое центростремительное ускорение?

A. 0 м / с 2

B. 2,5 м / с 2

H,50 м / с 2

G. 250 м / с 2

D.2 м / с 2

18. С каким периодом должна вращаться карусель радиусом 6,4 м, чтобы центростремительное ускорение человека на карусели было 10 м / с2 ?

A. 5 с

B. 0,6 с

B.16 с

G. 4 с

D. 2,5 с

19. Максимальное ускорение, с которым автомобиль может двигаться на поворотах, составляет 4 м / с2. … Каков минимальный радиус круга, по которому автомобиль может двигаться по горизонтальному участку дороги со скоростью 72 км / ч?

А.18 м

B. 1300 м

H 5 м

G. 100 м

20. Человек идет со скоростью 5 км / ч относительно вагона против направления его движения, поезд движется на скорость 20 км / ч относительно Земли. С какой скоростью движется человек относительно Земли?

A. 5 км / ч

B. 20 км / ч

H.25 км / ч

G. 15 км / ч

21 Силы F 1 и F 2, приложенные к одной точке тела, угол между векторами F1 и F 2 равен 90 0… Каков модуль равнодействующей этих сил?

A. F 1 — F 2

B. F 2 — F 1

B. F 1 + F 2

22. Сила притяжения действует на тело со стороны Земли. Какое из следующих утверждений верно для силы, действующей от этого тела на Землю?

A. F 2 = F 1

B. F 2

B. F 2 = 0

D. F 2>> F 1

D. F 2 = — F 1

23 В каких системах отсчета удовлетворяются все 3 закона механики Ньютона?

А.Только в инерциальных системах

B. Только в неинерциальных системах

B. В инерциальных и неинерциальных системах

D. В любой системе отсчета

24. Какая из следующих единиц является единицей измерения работы?

А. Джоуль

Б. Ватт

Вт. Ньютон

Г. Паскаль

D. Килограмм

25. Какая физическая величина в международной системе (СИ) измеряется в ваттах?

A. Прочность

B. Масса

B.Работа

G. Мощность

E. Давление

26. Наклонная плоскость дает 5-кратный прирост силы. Какое увеличение или уменьшение расстояния?

A. 5-кратный проигрыш

B. 5-кратный проигрыш

B. Не дает ни выигрыша, ни проигрыша

D. Победа или поражение в зависимости от скорости движения

27. По льду скользит фигурист весом 70 кг. Какая сила трения действует на фигуриста, если коэффициент трения коньков, скользящих по льду, равен 0,02?

A. 0,35 N

B.1.4 N

H. 3.5 N

G. 14 N

28 Спортсмен стреляет из лука в цель: Гравитация действует на стрелу:

A. когда спортсмен натягивает лук

B. когда стрела находится в рейс

Б. при попадании стрелы в цель

г. при всех этих положениях

29. Плот равномерно плывет по реке со скоростью 1,6 м / с. Человек идет по плоту в обратном направлении со скоростью 1,2 м / с. Какова скорость человека в береговой системе сообщений?

А.2,8 м / с

B. 1,2 м / с

V. 1,6 м / с

G. 0,4 м / с

30. Что такое единица измерения силы?

А. Джоуль

Б. Кулон

В. Ньютон

Г. Кельвин

31. Какая физическая величина является вектором?

A. Масса

B. Путь

B. Время

G. Прочность

32. Какая единица измерения мощности?

А. Герц

Б. Ватт

В. Генри

Г.Фарад

Секция 2 «Молекулярная физика»

33. Приводились в контакт два тела с разной температурой. Теплообмен между ними:

А. невозможен

Б. возможен только с другими дополнительными условиями

условия

В. возможен без каких-либо дополнительных

D. нет правильного ответа

34. Если поставить огурец в соленой воде, тогда через время он станет соленым. Выберите явление, которым вам обязательно придется воспользоваться при объяснении этого явления:

A.диффузия

B. конвекция

B. химическая реакция

G. теплопроводность

35. При какой температуре молекулы могут покидать поверхность воды?

A. только при температуре кипения

B. только при температуре выше 100 ° C

B. только при температуре выше 20 ° C

G. при любой температуре выше 0 ° C

36. Температура газа 250 K. Средняя кинетическая энергия молекул газа равна:

A. J

B.J

W.J

G. J

37. Когда надутый и завязанный узел воздушный шар был вынесен на улицу в морозный день, он уменьшился в размерах. Это можно объяснить:

A. уменьшился размер молекулы

B. кинетическая энергия молекул уменьшилась

B. количество молекул уменьшилось

G. молекулы распались на атомы

38. При разработке нового автомобиля, необходимо решить следующую экологическую проблему:

A.увеличить мощность двигателя

B. снизить токсичность выхлопных газов

B. повысить комфорт салона

г. снизить расход топлива

39. Температура первого корпуса 5 ° C, второго 260K, а третья 20 ° C. Каков правильный порядок перечисления этих тел в порядке возрастания температуры?

A. 1, 2, 3

B. 3, 2, 1

B. 2, 1, 3

G. 1, 3, 2

40. Увеличение содержания углекислого газа в земная атмосфера является следствием работы:

А.атомные электростанции

B. тепловые электростанции

V. гидроэлектростанции

D. электростанции любого типа

41. Где количество молекул больше: в одном моль водорода или в одном моле воды?

A. то же

B. в одном моль водорода

B. в одном моль воды

D. недостаточно данных для ответа

42. Какой ученый первым экспериментально определил скорость молекул:

А.Ломоносов

Б. Больцман

В. Эйнштейн

Г. Штерн

43. Где больше всего молекул: в одном моле кислорода или в одном моле ртути?

А. Одинаков

B. В кислороде больше

B. Ртуть содержит больше

D. Недостаточно данных для ответа.

44. Выразите температуру в Кельвинах 1000 C?

A. 100 K

B. 0 C

H. 373 K

G. 273 K

45. Когда два тела соприкасаются с разными температурами, теплообмен между ними

A.Возможно

B. Невозможно

B. Возможно при дополнительных условиях

D. Отсутствие данных

Раздел 3 «Электричество»

46. Какая из формул выражает закон Кулона:

47. Действующая сила на заряде 0,00002С в электрическом поле равна 4Н. Напряженность поля в этой точке:

A. 200000N / C

B. 0,00008N / C

B. 0,00008C / N

G. 5 · 10 -6 C / N

48. Источник тока с ЭДС 18 В имеет внутреннее сопротивление 30 Ом.Какое значение будет иметь сила тока при подключении к этому источнику резистора 60 Ом:

A. 0,9 A

B. 0,6 A

B. 0,4 A

G. 0,2 A

49. Какое утверждение (согласно к рисунку)

правильное. :

A. Частицы 1 и 2 отталкиваются, Частицы 2 и 3 притягиваются, Частицы 1 и 3 отталкиваются

B. Частицы 1 и 2 притягиваются; частицы 2 и 3 отталкиваются, частицы 1 и 3 отталкиваются

B.частицы 1 и 2 отталкиваются; частицы 2 и 3 притягиваются, частицы 1 и 3 притягиваются

D. Частицы 1 и 2 притягиваются, частицы 2 и 3 отталкиваются, частицы 1 и 3 притягиваются

50. Как изменится сила кулоновского взаимодействия двух точек электрические заряды изменятся, если расстояние между ними увеличить в 3 раза?

A. увеличится в 3 раза

B. уменьшится в 3 раза

V. увеличится в 9 раз

G. уменьшится в 9 раз

52.Единица измерения электрического заряда в системе:

A. кулон

B. браслет

B. ожерелье

G. амулет

53. Какова сила тока в резисторе 2 Ом, если напряжение на его концах равно 2 В:

A. 2 A

B. 1 A

B. 4 A

G. 1,5 A

54. Какие носители электрического заряда создают ток в жидкостях:

A. Электроны

B Ионы

В. дырки

Г. любые заряженные частицы

55.При напряжении 20 В через нить накала электрической лампы протекает ток 5 А. Сколько тепла будет выделять нить лампы за 2 минуты.

A. 2400 Дж

B. 12000 J

H. 200 J

G. 40 J

56. Как узнать, что в данной точке пространства существует электрическое поле?

A. поместите магнитную стрелку в эту точку и посмотрите, ориентирована ли она.

B. поместите заряд в эту точку и посмотрите, действует ли на нее сила электрического поля.

B. поместите лампу накаливания в эту точку и посмотрите, загорится ли она.

G. это невозможно определить экспериментально, так как поле не влияет на наши чувства

57. Какая единица измерения электрической емкости:

А. литр

Б. м 3

В. Фарад

Г. килограмм

58. Как конденсатор обозначается в электрических цепях:

59. В цепи протекает ток 3А при напряжении 300В. спираль электроплитки.Сколько энергии потребляет плитка за 15 секунд?

A. 450J

B. 2000J

H. 13500J

G. 9000J

60.B электрический чайник при нагревании воды происходит преобразование:

A. электрическая энергия в кинетическую энергию

B. энергия в электрическую энергию

B. электрическая энергия во внутреннюю энергию

G. внутренняя энергия в кинетическую энергию

61. Сопротивление резистора увеличено вдвое. Как изменился ток, протекающий через этот резистор?

А.уменьшилось в 2 раза

B. увеличилось в 2 раза

V. не изменилось

G. увеличилось в 4 раза

62. Носителями тока в металлах являются:

A. ионы

B. электроны

B. отверстия

D. любые заряженные частицы

63. Назовите единицу измерения силы тока:

A. ньютонов

B. :

A. Процесс протекания тока в жидкостях

B.процесс протекания тока в газах

B. процесс протекания тока в вакууме

G. удар молнии

65. Какие заряженные частицы переносят электрический ток в полупроводниках?

A. электроны и ионы

B. электроны и дырки

B. нейтроны

G. только ионы

66. От чего зависит сопротивление проводника?

A. температура

B. Размеры

B. Материал

D. Напряжение

67.Какой прибор используется для измерения сопротивления?

A. Омметр

B. Ваттметр

B. Амперметр

G. Динамометр

Раздел 4 «Магнитное поле»

68. На каком рисунке правильно изображены линии магнитной индукции вокруг проводника с током, направленным перпендикулярно самолет рисунка у нас?

69. Какая формула соответствует силе Ампера:

70. Явление получения электрического тока с использованием магнитного поля называется

A.Магнитная индукция

B. электрическая индукция

B. электромагнитная индукция

G. индукция

71. Как проходят линии магнитной индукции вокруг постоянного магнита?

72. Какая сила действует на заряженную частицу, движущуюся в магнитном поле:

А. Сила Ампера

Б. сила Архимеда

Б. Кулоновская сила

Г. Сила Лоренца

73. Что такое физическое количество 1 вебер?

A. Магнитная индукция

B.магнитный поток

B. индуктивность

D. ЭДС индукции

74. Когда постоянный магнит вставлен в катушку, в ней генерируется электрический ток. Как называется это явление?

A. электрическая индукция

B. магнитная индукция

B. самоиндукция

G. электромагнитная индукция

75. Какова энергия магнитного поля катушки с индуктивностью 2 H при токе сила 200 мА?

А.400 Дж

Б. 0,04 Дж

Н. 40 Дж

Г. 100 Дж

76. Какова физическая ценность 1 Тесла?

A. Магнитная индукция

B. Магнитный поток

B. Индуктивность

D. Индукция ЭДС

77. Действует проводник с током в магнитном поле:

A. Сила Лоренца

B. Амперная сила

B. Кулоновская сила

G. Сила Архимеда

78. На каком явлении основана работа трансформатора?

А.электромагнитная индукция

B. самоиндукция

B. индуктивность

G. инерция

79. Какое правило можно использовать для определения направления линии магнитной индукции вокруг проводника с током?

A. Правило левой руки

B. Правило правой руки

B. Правило Ленца

D. Правило смещения

Раздел 5 «Колебания и волны»

80. Какие колебания не существуют?

A. автоколебание

B.вынужденные колебания

Б. гармонические колебания

Г. автоколебания

81. От чего зависит скорость распространения волн?

A. от его длины

B. от его частоты

V. от его амплитуды

G от плотности среды

82. Что такое длина волны?

A. — расстояние от начала до конца волны

B. — расстояние между двумя соседними горками

B. — расстояние от вершины качелей до низа

G.- расстояние между точками, фазы которых различаются на π / 2

83. Период колебаний называется:

A. время одного колебания

B. количество колебаний за 1 секунду

B. наибольшее отклонение тела из положения равновесия

D. периодическое изменение положения тела в пространстве

84. С какой скоростью распространяются электромагнитные волны?

A. 300000 м / с

B. 300000 км / с

H. 314 м / с

G. 3,14 км / ч

85.Какая из следующих формул определяет формулу Томсона?

Раздел 6 «Оптика»

86. Луч света, падающий на поверхность воды, преломляется. Преломление светового луча объясняется тем, что:

A. скорость света в воде меньше, чем его скорость в воздухе

B. скорость света в воде больше, чем его скорость в воздухе

B. фотоны светового луча притягиваются молекулами воды

D. Фотоны светового луча отталкиваются молекулами воды

87 В туалете висят две куртки.Один синий, а другой желтый. Разные цвета курток указывают на то, что:

A. синяя куртка на ощупь холоднее желтой

B. синяя куртка лучше согревает

B. краски на куртках поглощают свет волны разной длины

G. желтая куртка прочнее

88. Сколько времени требуется свету, чтобы преодолеть расстояние от Земли до Луны, равное 400 000 км?

A. 0 с

B. 1,3 * 10-3 с

B 0,5 с

G.1,3 с

D. 1200 с

E. 8,3 мин

89. Угол падения луча на поверхность зеркала 200

A. 70 0

B. 80 0

H 40 0 ​​

G. 20 0

D. 90 0

90. Расстояние до точки обзора человека составляет 50 см. На каком расстоянии от зеркала он должен находиться, чтобы лучше рассмотреть свое отражение в зеркале?

A.50 см

B. 1 м

H 25 см

D. 12,5 см

D.Как можно ближе.

91. Объект находится на расстоянии 2 м от собирающей линзы с фокусным расстоянием 1 м. На каком расстоянии от объектива находится изображение?

A. 0,5 м

B. 1,5 м

H. 2 м

G. 1 м

E. Нет изображения

92. Оптическая система глаза строит изображение удаленных объектов перед сетчатка. Что это за дефект зрения и какие линзы нужны для очков?

А. Дальнозоркость, коллекционирование.

B. Дальнозоркость, рассеивание

B. Близорукость, собирательство.

D. Близорукость, рассеяние

93. Сколько времени требуется свету, чтобы преодолеть расстояние от Земли до Солнца, равное 150 миллионам км?

A. 0 с

B. 1,3 * 10 -3 с

B 0,5 с

G. 1,3 с

D. 1200 с

E. 8,3 мин

94. Дистанция наилучшего обзора человек 40 см. На каком расстоянии от зеркала ему нужно находиться, чтобы лучше рассмотреть свое отражение в зеркале?

А.10 см

B. 20 см

H 40 см

G. 80 см

D. Как можно ближе.

95. Угол падения луча на поверхность зеркала 700. … Какой угол между отраженным лучом и поверхностью зеркала?

A. 70 0

B. 80 0

H 40 0 ​​

G. 20 0

D. 90 0

96. Что называется дисперсией?

A. Огибание препятствий

B. Сложение двух световых волн

B.Зависимость показателя преломления от длины волны света

D. Изоляция одной волны от луча света

97. Какие явления доказывают, что свет представляет собой поток частиц?

A. Поляризация

B. Дисперсия

B. Фотоэффект

D. Дифракция

Раздел 7 «Атомная и ядерная физика»

98. Лазерное излучение:

A. Тепловое излучение

B. Вынужденное излучение

Б. спонтанное (спонтанное) излучение

г.люминесценция

99. Кто открыл явление фотоэлектрического эффекта:

A.M. Планк

Б.А. Эйнштейн

В.П. Лебедев

Столетов Г.А.

100. Определить энергию фотонов для света с частотой Гц.

A. J

B. J

WJ

D. данных в задаче недостаточно

101. Ядро содержит:

A. 100 протонов и 250 нейтронов

B. 250 протонов и 150 электронов

Б.100 протонов и 150 нейтронов

G. 250 нейтронов и 100 электронов

102. Электромагнитное излучение попадает на никелевую пластину, энергия фотонов которой составляет 8 эВ. В этом случае в результате фотоэффекта электроны вылетают из пластины с максимальной энергией 3 эВ. Какова работа выхода электронов из никеля?

A. 11 эВ

B. 5 эВ

V. 3 эВ

H. 8 эВ

103. В результате α-распада ядро ​​изотопа золота превращается в ядро:

104 .При строительстве АЭС необходимо решить следующую экологическую задачу:

A. снижение затрат на строительство

B. предотвращение радиоактивных выбросов в атмосферу

B. уменьшение габаритов ядерного реактора

D. Оценка запасов делящегося материала

105. Солнце испускает все виды электромагнитного излучения, унося каждую секунду энергию, равную J. На сколько килограммов масса Солнца уменьшается каждую секунду?

А.за кг

Б. за кг

В. за кг

г. за кг

106. Никелевая пластина освещается светом, энергия фотонов которого составляет 8 эВ. В результате фотоэффекта из пластины вылетают электроны с кинетической энергией 3,5 эВ. Какова работа выхода электронов из никеля?

A. 11,5 эВ

B. 4,5 эВ

V.

Добавить комментарий

Ваш адрес email не будет опубликован. Обязательные поля помечены *